download scanned papers here - Paper 1 Paper II


UPSC PRELIMS 2018 PAPER I -- ANSWER KEYS




BIRD'S EYEVIEW


  • 2018, a challenge at least as big as 2017, if not bigger : The Prelims for 2018 proved to be a big challenge to anyone – fresh or experienced – who either expected a set pattern based on the year 2016 or even the tough pattern of 2017, rather than being prepared for anything and everything – remember that “uncertainty is the key theme in major competitive exams”. The 2017 papers (both I and II) were not only tough, they were twisted, some questions were ambiguous and required many esoteric facts to crack. This affected the cutoffs. And for 2018, the paper was factually tough, though not necessarily as analytically contorted as was the case for 2017. Lesson (once more) : Long term tapasya, going in-depth, no keep hitting more, discretion, practice.

  • Summary of major changes :
    1. Topicwise balance same in some areas – (a) International CA and Indian CA (9 and 7 questions in 2018, compared to 8 and 7 in 2017), (b) Environment and Ecology (11 questions compared to 12 for 2017), (c) Economy of India (11 compared to 8 in 2017)
    2. Topicwise balance changed in some areas – (a) Science and Technology including Agriculture (more than doubled to 14 questions compared to 6 asked in 2017), (b) Government schemes reduced to half (6 instead of 12 last year), (c) Modern history more than doubled (13 in place of 7 asked in 2017), (d) Polity dropped drastically (12 in place of 22 asked in 2017), and (e) Art and Culture went up (6 questions compared to 2 last year)
    3. Toughness level remained more or less the same as in 2017, but the analytical touch (as in 2017) was missing. It was intense on the factual toughness part.
    4. Lot of micro level stuff asked (just like in 2017)
    5. Closely placed options in some questions, making guesstimates a risky game
    6. Trying to “hit more” may backfire
    7. Only long term prep helped – no mugging up of “lists”
    8. Total number of sitters that all students must crack : 11 questions. In Set A, these were questions numbered 6 (MDR), 7 (NSG-NPT), 15 (ATM-NPCI), 18 (Yemen-Sudan), 24 (Two state solution), 26 (GIS-WTO), 43 (THAAD), 58 (Petya), 66 (IoT), 90 (Catalonia), and 92 (Right to Privacy). No student should have missed these ones.
  • Few direct questions : Since 2016 and 2015 were current affairs heavy, and many questions could be solved ONLY by elimination of options, and since many questions were very direct requiring little more than having remembered (or even read) the concept earlier, 2017 was a rude jolt. The 2018 paper continued the streak. If you were prepared for this kind of stuff, the overall composure would have helped. Lesson : It is only a relative performance test, not absolute.

  • Topicwise balance changed : The paper was not dominated by questions from any one area unlike in 2017 when Polity had 22 questions and Government Schemes 12 questions. Heaviest this year were : Science and Technology including Agriculture (14 questions), 13 from Modern History, 12 from Polity, and 11 each from Environment & Ecology and Economy of India. Lesson : Don’t expect anything.

  • Easiness limited to the sitters : The 11 questions mentioned above were easy.

  • Government schemes limited : Not too many questions, unlike in 2017.

  • India's Independence struggle was given unusual importance this year (13 / 7).

  • Our Support : At PT's IAS Academy, we have tried to incorporate everything possible on as many topics as possible. We wish our students great luck in the results!

  • Ambiguous answers : Unlike 2017, there were no major ambiguities, but some possible issues of interpretation. Year 2017 had questions like those on Butterfly / Vote / IVC / gypsum etc. This is good news.

  • At this stage, we would like to suggest students to avoid getting into any speculation regarding the "Cut-Offs" for Prelims. A broad guideline could be General Category – 95 to 102 Marks, OBC – almost same and SC/ST – 85 to 92 marks. If you are anywhere close to these, get cracking for Mains preparations. Do not sulk, do not lose hope. Strange things can happen. All the best!



TOPICWISE DISTRIBUTION




DETAILED QUESTIONWISE SOLUTIONS (Set A)


1. Consider the following statements :

  1. In the first Lok Sabha, the single largest party in the opposition was the Swatantra Party.
  2. In the Lok Sabha, a "Leader of the opposition" was recognised for the first time in 1969.
  3. In the Lok Sabha, if a party does not have a minimum of 75 members, its leader cannot be recognised as the Leader of the Opposition.

Which of the statements given above is/are correct?

  1. 1 and 3 only
  2. 2 only
  3. 2 and 3 only
  4. 1, 2 and 3

Sol. Answer is (b). Only statement 2 is correct.

Statement 1 is wrong as the Swatantra Party was an Indian liberal-conservative political party that existed from 1959 to 1974, founded by C. Rajagopalachari in reaction to what he felt was Nehru's INC's increasingly socialist thinking (Fabian socialism). The Congress (INC) won a landslide victory, in 1951-52 elections winning 364 of the 489 seats and 45% of the total votes polled. This was more than four times as many votes as the second-largest party. Nehru became the first PM.

Statement 3 is wrong as the rules state that any party needs to bag at least 10% of the total seats for its leader to claim the status of the leader of the opposition. Out of 545 (543 + 2), a party needs at least 55 seats.Though having the largest contingents among the opposition, Congress with 44 members fell short of the number in May-June 2014 (and onwards). The situation is the same as it was for 10 years from August 1979 to December 1989 — the 7th and 8th Lok Sabhas.

Statement 2 is correct as the first time a leader of the opposition was appointed, was in 1969 when Ram Subhag Singh of the Congress (Organisation) was recognised. This was after Congress split post expulsion of Indira Gandhi.

This was a tough question, and required micro knowledge of a topic in news recently. Can view support images here

Here are the support images.


2. Which of the following leaf modifications occur(s) in the desert areas to inhibit water loss?

  1. Hard and waxy leaves
  2. Tiny leaves
  3. Thorns instead of leaves

Select the correct answer using the code given below:

  1. 2 and 3 only
  2. 2 only
  3. 3 only
  4. 1, 2 and 3

Sol. Answer is (d). All modifications mentioned happen.

Deserts cover about one fifth of the earth's land surface. Desert plants save water in order to merely survive! During photosysnthesis to make sugar, the stomata on a plant's leaves and stems open to absorb CO2 from the air and in return release oxygen. Each time a plant does so, some water escapes in a process called "transpiration". Replacing it is not easy with little rainfall, so desert plants have acquired special adaptations. Examples are :

(1) Waxy cover : Plants not only lose water through their pores, they also lose it through the cell walls on their leaves. The leaves and stems of many desert plants have a thick covering that is coated with a waxy substance, allowing them to seal in the moisture.

(2) Tiny leaves : Most desert plants have small leaves, spikes for leaves, or no leaves at all. The smaller or fewer leaves a plant has, the less water is lost during transpiration since it has less surface area open to the sun and wind. The twigs and stems help to carry out photosynthesis in such cases.

(3) Smaller, fewer, and deeper pores : With such pores, hot and dry winds are prevented from blowing directly across the pores and evaporating so much of the plants' water.

(4) Nocturnal : Plants lose a large amount of water through transpiration, especially on sunny days but if the process occurs at night, less water is lost.

This question from Geography (and Environment Ecology) required some basic knowledge.

Liked Analysis? Write a lovely Testimonial! Thanks :)


3. As per the NSSO 70th Round “ Situation Assessment Survey of Technical Households” consider the following statements:

  1. Rajasthan has the highest percentage share of agricultural households among its rural households.
  2. Out of the total agricultural households in the country, a little over 60 percentage belongs to OBCs.
  3. In Kerala, a little over 60 percent agricultural households reported to have maximum income from sources other than agricultural activities.

Which of the statement given below is/ are correct?

  1. 2 and 3 only
  2. 2 only
  3. 1 and 3 only
  4. 1, 2 and 3

Sol. Answer is (c). as statement 2 is wrong.

OBCs are less than 50% of the total, as per the 2013 survey. This is a tough question, as micro data from the NSSO 70th SASAH (Situation Assessment Survey of Agri Households), 2013 is being asked.

Statements 1 and 3 are correct.

You can download the entire PDF report here . View the support images here.


4. How is the National Green Tribunal (NGT) different from Central Pollution Control Board (CPCB)?

  1. The NGT has been established by an Act and CPCB has been created by an executive order of the government.
  2. The NGT provides environmental justice and helps reducing the burden of litigation in the High Courts whereas the CPCB promotes the cleanliness of streams and wells , aims to improve the quality of air in the country.
  3. In Kerala, a little over 60 percent agricultural households reported to have maximum income from sources other than agricultural activities.

Which of the statement given below is/are correct?

  1. 1 only
  2. 2 only
  3. Both 1 and 2
  4. Neither 1 nor 2

Sol. Answer is (b). Only statement 2 is correct.

Both are established by Acts, hence Statement 1 is incorrect.

The National Green Tribunal was established on 18.10.2010 under the National Green Tribunal Act 2010 for effective and expeditious disposal of cases relating to environmental protection and conservation of forests and other natural resources including enforcement of any legal right relating to environment and giving relief and compensation for damages to persons and property and for matters connected therewith or incidental thereto. It is a specialized body equipped with the necessary expertise to handle environmental disputes involving multi-disciplinary issues.

The Central Pollution Control Board of India is a statutory organisation under the Ministry of Environment, Forest and Climate Change. It was established in 1974 under the Water Act, 1974.

Hence, both are established by Acts, and statement 1 is wrong.

Regarding statement 2, use source : http://cpcb.nic.in/Introduction/ : "It serves as a field formation and also provides technical services to the Ministry of Environment and Forests of the provisions of the Environment (Protection) Act, 1986. Principal Functions of the CPCB, as spelt out in the Water (Prevention and Control of Pollution) Act, 1974, and the Air (Prevention and Control of Pollution) Act, 1981, (i) to promote cleanliness of streams and wells in different areas of the States by prevention, control and abatement of water pollution, and (ii) to improve the quality of air and to prevent, control or abate air pollution in the country."

Hence, answer is (b) - 2 only.




5. Consider the following statements:

  1. The Parliament of India can place a particular law in the Ninth Schedule of the Constitution of India
  2. The validity of law placed in Ninth Schedule cannot be examined by any court and no judgement can be made on it.

Which of the statements given below is/ are correct?

  1. 1 only
  2. 2 only
  3. Both 1 and 2
  4. Neither 1 nor 2

Sol. Answer is (a). Statement 2 is wrong.

Nehru government's First Amendment in 1951 along with Article 31-B marked the addition of the Ninth Schedule to the Constitution. It was intended to protect land reform laws from being challenged in courts on the grounds of violation of fundamental rights. According to Article 13(2) of the Constitution, the state shall not make any law that will be inconsistent with the fundamental rights, and such laws shall be void to the extent of the contravention.

Once a law is enacted and included in the Ninth Schedule, it gets protection under Article 31-B (validation of certain Acts and Regulations) and is not subject to judicial scrutiny. Over the years 284 laws were included in it and about 30 of them are under challenge.

In January 2007, the Supreme Court after examining the scope and powers of Parliament to enact laws and include them in the Ninth Schedule, held that the power of judicial review could not be taken away by putting a law under the Ninth Schedule. The SC then said : "The power to grant absolute immunity at will is not compatible with the basic structure doctrine and, therefore, after April 24, 1973 the laws included in the Ninth Schedule would not have absolute immunity. The validity of such laws can be challenged on the touchstone of basic structure such as reflected in Article 21 read with Article 14 and Article 19, Article 15 and the principles underlying these Articles. Insertion in the Ninth Schedule is not controlled by any defined criteria or standards by which the exercise of power may be evaluated. The consequence of the insertion is that it nullifies entire Part III (relating to fundamental rights) of the Constitution."


6. Which of the following best describes the term “ Merchant Discount Rate” sometimes seen in news ?

  1. The incentive by a Bank given to a merchant for accepting payments through debit cards for financial transactions for purchasing goods and services.
  2. The amount paid by bank to their customers when they pay by debit cards for financial transactions for purchasing goods and services.
  3. The charge to a merchant by a bank for accepting payments from his customers through the bank’s debit card.
  4. The incentive given by the Government to merchants for promoting digital payments by their customers through PoS( point of sale) machines and debit cards.

Sol. Answer is (c).

MDR is a fee charged from a merchant by a bank for accepting payments from customers through credit and debit cards in their establishments. In December 2017, the government said it would reimburse the merchant discount rate (MDR) applicable on digital transactions up to Rs.2,000 for the next two years, to promote the cashless culture.

This was a "sitter", especially in the post-demonetisation era, and you must have cracked it :)


7. Which is /are the consequence / consequences of becoming a member of the ‘Nuclear supplier Group’?

  1. It will have access to the latest and most effective nuclear technologies.
  2. It automatically becomes a member of “ The Treaty of non- proliferation of Nuclear weapons(NPT)”

Which of the statements given below is/ are correct?

  1. 1 only
  2. 2 only
  3. Both 1 and 2
  4. Neither 1 nor 2

Sol. Answer is (a).

Becoming a member of the NSG has no relation with automatic membership of the NPT regime. India has not signed the NPT as it wants worldwide nuclaer disarmanent first, but India wants to get into the NSG.

Hence, statement 2 could not be right at all. But 1 is indeed correct. A hot topic, and an easy question!

This was a sitter, as you could asses the Indian situation.

Here are the support images.


8. With reference to India’s decision to levy an equalization tax of 6% on online advertisement services offered by non resident entities , which of the following statement is/are correct?

  1. It is introduced as a part of Income Tax Act.
  2. Non Resident entities that offer advertisement services in India can claim a tax credit in their home country under the “ Double Taxation Avoidance commission”

Select the correct answer using the code given below:

  1. 1 only
  2. 2 only
  3. Both 1 and 2
  4. Neither 1 nor 2

Sol. Answer is (d).

The levy was ntroduced in the Budget 2016-17 as part of the finance bill and not as a part of Income Tax Act. So, because of this the companies would not be able to take the benefit of tax treaties to avoid double taxation in their home countries.

The term “Equalization Levy” (or Google Tax) made its first appearance in budget 2016-17 documents. The Government put a 6% equalization levy on the income accrued to a foreign E-commerce company which is not a resident of India (no Permanent Establishment here). This affected Google, Amazon, Facebook etc.

The trick in the question was to remember that this tax was part of the Union Budget, and not of the Income Tax Act. That would make both 1 and 2 wrong.


9. Consider the following Statements:

  1. The Fiscal responsibility and the budget Management Review Committee Report has recommended a debt to GDP ratio of 60% for the general (combined) Government by 2023, comprising 40% for the Central government and 20% for the State Government.
  2. The Central government has domestic liabilities of 21% of GDP as compared to 49% of GDP of the State Governments.
  3. As per the Constitution of India, it is mandatory for the state to take Central government’s consent for raising any loans if the former owes any liabilities to the latter.

Which of the given statement is/ are correct?

  1. 1 only
  2. 2 and 3 only
  3. 1 and 3 only
  4. 1, 2 and 3

Sol. Answer is (c).

The FRBM Review Committee Report 2017 had to figure in the exam!

Statement 1 is correct. That is the core recommendation.

Statement 2 is wrong as the numbers are interchanged (21% and 49% for States and Centre).

Statement 3 is technically correct.

The Constitution of India has adopted a federal system of polity and governance, originally envisaging a two-tier structure: Central (i.e., federal) Government and State (i.e., provincial) Governments. With the Constitution (73rd and 74th) Amendment Acts, 1992, Local Self-Government Institutions (LSGIs), i.e., rural and urban local bodies have been accorded constitutional status as the third-tier of Government. The Constitution provides for preparation of annual budgets and borrowings by the Centre, respectively, under Articles 112 and 292 and by the States under Articles 202 and 293.

Article 293 of the Indian Constitution imposes certain restrictions on the borrowings by State Governments. The Article stipulates that a State may not, without the consent of the Government of India, raise any borrowings if it has any loan outstanding, which is repayable to the Government of India.

Here are support images.

Now, refer these PDF files for more.

FRBM Review Committee 2017 report - download here

RBI on rules of borrowing - download here

Enrol in The Hindu editorial analysis | Comment on this question & solution now!


10. Consider the following statements:

  1. The quality of imported edible oil is more than the domestic production of edible oils in the last five years.
  2. The Government does not impose any custom duty on all the imported edible oils as a special case.

Which of the statements given above is/are correct?

  1. 1 only
  2. 2 only
  3. Both 1 and 2
  4. Neither 1 nor 2

Sol. Answer is (a). Only statement 1 is correct.

India’s vegetable oil imports have skyrocketed in recent years to about 14 million tonnes, worth approximately $11 billion (over Rs.70,000 crore). In value terms vegoil imports are next only to crude and gold, and the highest for any food commodity. This is an alarming situation. Our import dependence has worsened to over 70 per cent (so we locally produce only 30% of our needs). Statement 1 is correct.

Statement 2 is wrong, as government does impose customs duty on imported edible oils. In the past decades, liberal policies have meant low rates, and at times zero rates too, but duties have persisted.

There was another distressed sector - Pulses - where the government demonstrated courage in taking steps with far reaching implications. Our domestic production has grown dramatically. But we have been accused of protectionist tendencies. There is immense international pressure to ease policy restrictions on pulses; yet, the priority at the moment is remunerative prices for pulse growers. Same should ultimately be the case with edible oils sector too.

Here are some support images!

Here is a support PDF file for download - here

Liked Analysis? Write a lovely Testimonial! Thanks :)


11. He wrote biographies of Mazzini, Garibaldi, Shivaji and Shrikrishna; stayed in America for sometime; and was elected to the Central Assembly. He was:

  1. Aurobindo Ghosh
  2. Bipin Chandra Pal
  3. Lala Lajpat Rai
  4. Motilal Nehru

Sol. Answer is (c).

This remarkable credit goes to none other than the great nationalist leader Lala Lajpat Rai!

Lajpat Rai travelled to the US in 1907, and then returned during World War I. He joined the 'Swaraj Party' of Motilal Nehru and was elected to the Central Legislative Assembly. By that time the Hindu Muslim unity move shaped by Mahatma Gandhi had failed. He wrote the biographies of the patriots Mazzini and Garibaldi who unified Italy. He also wrote outstanding books about Indian great men - Shivaji, Sri Krishna and Dayananda Saraswati. The books on Mazzini and Shivaji contained passages, which encouraged people to fight for freedom.

A support PDF - download here.


12. Consider the following statements:

  1. Aadhar card can be used as a proof of citizenship or domicile.
  2. Once issued, Aadhar card can not be deactivated or omitted by the Issuing Authority.

Which of the following statements given above is/are correct?

  1. 1 only
  2. 2 only
  3. Both 1 and 2
  4. Neither 1 nor 2

Sol. Answer is (d). Both given statements are wrong!

Aadhar card cannot (at present) be used as a proof of citizenship of India, but only as a proof of identity. Hence statement 1 is wrong.

Aadhar number can indeed be deactivated or omitted by the UIDAI. In August 2017, UIDAI reported it had deactivated 81 lac numbers already.

Conditions for deactivation can be :

(1) An Aadhaar with mixed or anomalous biometric information or multiple names in single name

(2) Non-usage of the same for three consecutive years - so non-linking with any bank account or PAN or for carrying out any transaction such as giving Aadhaar details to EPFO to claim pension can create trouble

(3) Children below 5 years need to update their details again after 15 years of age

A deactivated Aadhaar can be activated again upon re-verification of your biometrics or update of your card.


13. Which of the following has/have shrunk immensely/ dried up in the recent past due to human activities ?

  1. Aral Sea
  2. Black Sea
  3. Lake Baikal

Select the correct answer using the code given below:

  1. 1 only
  2. 2 and 3
  3. 2 only
  4. 1 and 3

Sol. Answer is (d). Both 1 and 3 are correct.

The Aral sea used to be one of the largest lakes in the world (area of 68,000 sqkm). But it has been shrinking since the 1960s after the rivers that fed it were diverted by Soviet irrigation projects. An example of massive changes brought indiscriminately! By late 1990s, the Aral Sea declined to less than 10% of its original size, splitting into four lakes – the North Aral Sea, the eastern and western basins of the once far larger South Aral Sea, and one smaller intermediate lake.

The Aral Sea had no outflow (endorheic lake) and lay between Kazakhstan and Uzbekistan. Work is being done to restore in part the North Aral Sea.

Lake Baikal is the largest freshwater lake by volume in the world, containing 23% of the world's fresh surface water. The world's largest freshwater lake saw its water level drop eight centimeters below the minimum level of 456 meters, and there are warnings of dire consequences ahead.

The Black Sea is a body of water and marginal sea of the Atlantic Ocean supplied by a number of major rivers. With the Black Sea no such shrinking is reported.

This was a tough question, and required micro knowledge of a topic in news recently. Can view support images here


14. “ Rule of the law Index “ is released by which of the following?

  1. Amnesty International
  2. International Court of Justice
  3. The Office of UN Commissioner of Human Rights
  4. World Justice Project

Sol. Answer is (d).

The WJP Rule of Law Index measures rule of law adherence in 113 countries and jurisdictions worldwide based on more than 110,000 household and 3,000 expert surveys. Featuring primary data, the WJP Rule of Law Index measures countries’ rule of law performance across eight factors: Constraints on Government Powers, Absence of Corruption, Open Government, Fundamental Rights, Order and Security, Regulatory Enforcement, Civil Justice, and Criminal Justice.

The top three overall performers in the 2017-2018 WJP Rule of Law Index were Denmark (1), Norway (2), and Finland (3); the bottom three were Afghanistan (111), Cambodia (112), and Venezuela (113). The top three and bottom three performing countries have not changed since the 2016 Index.

Globally, countries in Western Europe and North America continue to top the WJP Rule of Law Index, followed by countries in the East Asia and Pacific region. On average, the South Asia region scored lowest. that is food for some serious thought for all of us!

Support images can be viewed here

A support PDF file for download - here


15. Which one of the following links all the ATMs in India?

  1. Indian Bank’s Association
  2. National Securities Depository Ltd.
  3. National Payments Corporation of India
  4. Reserve Bank of India

Sol. Answer is (c).

The first ATM in India was set up in 1987 by HSBC in Mumbai. The National Financial Switch (NFS) connects 1,98,953 ATMs of 449 banks (91 Member Banks, 358 Sub- Member). This NFS is the largest network of shared automated teller machines (ATMs) in India. It was designed, developed and deployed by the Institute for Development and Research in Banking Technology (IDRBT) in 2004, with the goal of inter-connecting the ATMs in the country and facilitating convenience banking.

In 2009, the Board for Regulation and Supervision of Payment and Settlement Systems (BPSS) granted Certificate of Authorisation to the NPCI for operation of National Financial Switch (NFS) ATM Network.

National Payments Corporation of India has ten core promoter banks - State Bank of India, Punjab National Bank, Canara Bank, Bank of Baroda, Union Bank of India, Bank of India, ICICI Bank, HDFC Bank, Citibank and HSBC.


16. Consider the following statements:

  1. Capital Adequacy Ratio (CAR) is the amount that the banks have to maintain in the form of their own funds to offset any loss that bank incur if the account holder fail to repay any dues.
  2. CAR is decided by each individual bank.

Which of the following Statements given above is/are correct?

  1. 1 only
  2. 2 only
  3. Both 1 and 2
  4. Neither 1 nor 2

Sol. Answer is (d). Both statements are wrong!

The Capital Adequacy Ratio (CAR) is a ratio of a bank’s capital to its risk. It measures a bank's capital in relation to its risk-weighted assets.

Statement 2 is obviously wrong as RBI would decide it in India, while the Basel III norms are global ones, used by national regulators elsewhere. As of 2017, under Basel III, a bank's tier 1 and tier 2 capital must be at least 8% of its risk-weighted assets. The Capital Adequacy ratio (CAR) in India is 9 per cent.

Statement 1 is wrong as CAR is not an amount, but a numberless ratio (from the statement's wording "...CAR is the amount...). Coming to risk, it can be of various types - credit risk, market risk and operational risk.

The Statement 1 is referring only to credit risk, not the other two types. Hence 1 can be considered wrong.

Information : CAR is a ratio of a bank’s capital to its risk. This capital is further classified into two – Tier 1 (the main portion of the banks’ capital, usually in the form of equity shares) and Tier 2 capital. Out of the 9 per cent of CAR, 7 per cent has to be met by Tier 1 capital while the remaining 2 per cent by Tier 2 capital. Under Basel III, the minimum capital adequacy ratio that banks must maintain is 8%. The capital-to-risk-weighted-assets ratio promotes financial stability. The CAR is calculated by adding tier 1 capital to tier 2 capital and dividing by risk-weighted assets. Tier 1 capital is the core capital of a bank, which includes equity capital and disclosed reserves. This type of capital absorbs losses without requiring the bank to cease its operations; tier 2 capital is used to absorb losses in the event of liquidation.

You can view an RBI document - PDF download here


17. The identity platform ‘Aadhar’ provides open “Application Programming Interfaces(APIs)”. What does it imply?

  1. It is integrated into an electronic device.
  2. Online authentication using iris is possible.

Which of the statements given above is/are correct?

  1. 1 only
  2. 2 only
  3. Both 1 and 2
  4. Neither 1 nor 2

Sol. Answer is (c). Both statements are correct.

Statement 1 speaks about "any electronic device" so we can assume our mobile phones and other devices (attendance recorders etc.) are covered in this definition. Aadhar API can indeed be integrated.

Biometric devices can be :

(1) Discrete Devices: These type of devices refer to the class of biometric devices (Fingerprint/IRIS) that require connectivity to a host device such as PC/laptop/Micro ATM etc.

(2) Integrated Devices: The integrated devices have the sensor integrated into the device package i.e. phone/tablet etc.

The form factors in which biometric devices may be deployed include:

Hand-Held / PoS Device such as MicroATMs, attendance devices, USB device connected to PC, Mobile phone with biometric sensor, Kiosks such as ATMs, MNREGA job request kiosks.

Statement 2 is also correct. Type 3 from the following Aadhaar Authentication Offerings includes Iris. ( source : https://authportal.uidai.gov.in/home-articles?urlTitle=aadhaar-authentication-offerings&pageType=authentication )

Type 1 Authentication -Through this offering, service delivery agencies can use Aadhaar Authentication system for matching Aadhaar number and the demographic attributes (name, address, date of birth, etc) of a resident.

Type 2 Authentication -This offering allows service delivery agencies to authenticate residents through One-Time-Password (OTP) delivered to resident's mobile number and/or email address present in CIDR.

Type 3 Authentication -Through this offering, service delivery agencies can authenticate residents using one of the biometric modalities, either iris or fingerprint.

Type 4 Authentication -This is a 2-factor authentication offering with OTP as one factor and biometrics (either iris or fingerprint) as the second factor for authenticating residents.

Type 5 Authentication -This offering allows service delivery agencies to use OTP, fingerprint & iris together for authenticating residents.

18. Very recently, In which of the following countries having lakhs of people either suffered from severe famine/acute malnutrition or died due to starvation caused by war/ ethnic conflicts?

  1. Angola and Zambia
  2. Morocco and Tunisia
  3. Venezuela and Colombia
  4. Yemen and South Sudan

Sol. Answer is (d).

The question specifically talks about famine / malnutrition and wars etc. In option (b), Tunisia could qualify, but it was due to Arab Spring and not these reasons. In option (c), Venezuela could qualify, but it is due to their own internal political reasons. Option (d) suits best, as both Yemen (an intense war with Saudi Arabia), and South Sudan (continuing civil war and famile) fit the description given.

This question was a sitter, and should have been solved!

Liked Analysis? Write a lovely Testimonial! Thanks :)


19. Regarding Wood’s Dispatch, which of the following statements are true:

  1. Grants-in-Aid system was introduced.
  2. Establishment of universities was recommended.
  3. English as a medium of instruction at all levels of education was recommended.

Select the correct answer using the code given below:

  1. 1 and 2 only
  2. 2 and 3 only
  3. 1 and 3 only
  4. 1, 2 and 3

Sol. Answer is (a). Statement 3 is wrong.

Sir Charles Wood (President of the Board of Control of the English East India Company) in 1854 he sent a despatch to Lord Dalhousie (Governor-General of India) suggesting that primary schools must adopt vernacular languages, high schools must adopt Anglo vernacular language and on college level English medium for education. This is known as Wood's despatch. Hence statement 3 is wrong. That directly rules out 3 options - (b), (c) and (d). Only (a) is left!

Statements 1 and 2 are correct.

As a result of Wood's despatch, this happened :

(1) New institutions like the University of Calcutta, the University of Bombay and the University of Madras were set up in 1857; the University of the Punjab setup in 1882 and the University of Allahabad in 1887

(2) General promotion of English education in India.


20. With the reference to Parliament of India, which of the following Parliamentary Committees scrutinizes and reports to the House whether the power to make regulations, rules , sub-rules, by-laws, etc. conferred by the Constitution or delegated by the Parliament are being properly exercised by the Executive within the scope of such delegation?

  1. Committee on Government Assurances
  2. Committee on Subordinate Legislation
  3. Rules Committee
  4. Business Advisory Committee

Sol. Answer is (b). Statement 3 is wrong.

1. Subordinate legislation is the legislation made by an authority subordinate to the legislature. It proceeds from any authority other than the sovereign power and is, therefore, dependent for its continued existence and validity on some superior or supreme authority.

2. Most of the enactments provide for the powers for making rules, regulations, bye-laws or other statutory instruments which are exercised by the specified subordinate authorities. Such legislation is to be made within the framework of the powers so delegated by the legislature and is, therefore, known as delegated or subordinate legislation.

3. Subordinateness indicates the nature of the legislation itself. Delegated legislation under such delegated powers is ancillary and cannot, by its very nature, replace or modify the parent law nor can it lay down details akin to substantive law.

4. Rule 209 of the Rules of Procedure and Conduct of Business in Rajya Sabha lays down the functions of the Committee thus:

After each rule, regulation, bye-law, scheme or other statutory instrument (hereinafter referred to as the 'order') framed in pursuance of the Constitution or the legislative functions delegated by Parliament to a subordinate authority and which is required to be laid before Parliament, is so laid before the Council, the Committee shall, in particular consider:

(1) whether the order is in accord with the provisions of the Constitution or the Act pursuant to which it is made;

(2) whether the order contains matter which in the opinion of the Committee should more properly be dealt within an Act of Parliament;

(3) whether the order contains imposition of taxation;

(4) whether the order directly or indirectly bars the jurisdiction of the court;

(5) whether the order gives retrospective effect to any of the provisions in respect of which the Constitution or the Act does not expressly give any such power;

(6) whether the order involves expenditure from the Consolidated Fund of India or the public revenues;

(7) whether the order appears to make some unusual or unexpected use of the powers conferred by the Constitution or the Act pursuant to which it is made;

(8) whether there appears to have been unjustifiable delay in its publication or laying the order before Parliament; and

(9) whether for any reason the form or purport of the order calls for any elucidation.

Read more on this PDF download - here


21. Consider the following statements:

  1. As per the right to education (RTE) Act, to be eligible for appointment as a teacher in a state, a person would be required to possess the minimum qualification laid down by the concerned State council of Teacher education.
  2. As per the RTE Act, for teaching primary classes, a candidate is required to pass a Teacher Eligibility Test conducted in accordance with the National Council of Teacher Education guidelines.
  3. )In India, more than 90 % of teacher education institutions are directly under the State Governments.

Which of the statements given above is/are correct ?

  1. 1 and 2
  2. 2 only
  3. 1 and 3
  4. 3 only

Sol. Answer is (b). Only statement 2 is correct.

A question asked in the Parliament received this answer :

1. The Sarva Shiksha Abhiyan (SSA) does not prescribe any norms for the appointment of Teachers of Elementary Education.

2. However, the National Council of Teacher Education (NCTE), as the academic authority notified by the Central Government, under the Right of Children to Free and Compulsory Education (RTE), Act 2009, has laid down the minimum educational & professional qualifications for a person to be eligible for an appointment as a teacher for classes I-VIII (primary classes - statement 2 of our question)

3. These are applicable to all schools imparting elementary education, including the schools under the State Governments and to qualify under a Teacher Eligibility Test (TET).

4. Some States are finding it difficult to appoint teachers due to the non- availability of qualified candidates because of the inadequate number of teacher education institutions and due to delays in the Teacher Eligibility Tests (TET) because of court cases.

5. A total of 13 States had requested a relaxation in the professional qualifications as laid down by the NCTE, which has been duly considered and permitted by the Central Government under certain conditions. Some States had also sought relaxation from the TET, which has not been agreed to.

A tough one, asking the micro details of the process.

Can read a support PDF - here


22. Consider the following pairs:

Which of the pairs given above is/are correct?

  1. 1 only
  2. 1 and 2
  3. 1 and 2
  4. 2 and 3

Sol. Answer is (b).

Statement 1 is correct. The Chapchar Kut is a festival of Mizoram, India, celebrated during March after completion of Jhum operation i.e., jungle-clearing (clearing of the remnants of burning). It is a spring festival, estimated to have started in 1450-1700 A.D. in a village called Suaipui. The festival is observed in the last part of February or the early part of March when the trees and bamboos felled for jhum are left to dry and the shifting cultivators have time to enjoy.

Statment 2 is correct. Khongjom Parba is a style of ballad singing from Manipur using Dholak (drum) which depicts stories of heroic battle fought by Manipuris against the mighty British Empire in 1891. It is one of the most popular musical art forms of Manipur inciting the spirit of patriotism and nationalism among the people at one time. With declining patronage and listeners, performance of Khongjom Parva has declined drastically thereby compelling the indigenous musical art form to disappear gradually from the minds of the people. The future of Khongjom Parva traditional music is uncertain after the death of the few existing veteran singers.

Statement 3 is wrong. Huyen langlon is an Indian martial art from Manipur. In the Meitei language, huyen means war while langlon or langlong can mean net, knowledge or art. Huyen langlon consists of two main components: thang-ta (armed combat) and sarit sarak (unarmed fighting).


23. Consider the following statements :

  1. The Food Safety and Standards Act, 2006 replaced the Prevention of Food Adulteration Act, 1954.
  2. The Food Safety and Standard Authority of India (FSSAI) is under the charge of Director General of Health Services in the Union Ministry of Health and Family Welfare.

Which of the following statements is/are correct ?

  1. 1 only
  2. 2 only
  3. Both 1 and 2
  4. Neither 1 and 2

Sol. Answer is (a). Statement 2 is wrong.

According to the Food Safety and Standard Act (Chapter XII), any reference to “the Prevention of the Food Adulteration Act, 1954” shall be substituted by it.

The following come under the Directorate General Health Services : CBHI, CDSCO, Central Health Education Bureau, Leprosy Section, Medical store Organisation, MH, and International Health. Clearly, FSSAI is not included.

Statement 1 is correct.

You can download the PDF of FSS Act 2006 here

Download the HINDI version of the Act here

Detailed study material on FSSAI (Orange Book PDF here, Pink Book PDF here)


24. The term “ two state solution” is sometimes mentioned in the news in the context of the affairs of

  1. China
  2. Israel
  3. Iraq
  4. Yemen

Sol. Answer is (b).

The two state solution is a common way of referring to the solution of the ongoing crisis between Israel and Palestine. This question was a sitter, and should have been solved straightaway!

[ Total number of sitters that all students must crack : 11 questions. In Set A, these were questions numbered 6 (MDR), 7 (NSG-NPT), 15 (ATM-NPCI), 18 (Yemen-Sudan), 24 (Two state solution), 26 (GIS-WTO), 43 (THAAD), 58 (Petya), 66 (IoT), 90 (Catalonia), and 92 (Right to Privacy). No student should have missed these ones ]

Reference images here

Reference PDF for download - Israel-Palestine problem


25. With reference to the provisions made under the national Food Security Act, 2013, consider the following statements:

  1. The families coming under the category of ‘below poverty line (BPL)’ only are eligible to receive subsidised food grains.
  2. The eldest woman in a household, of age 18 years or above, shall be the head of the household for the purpose of issuance of a ration card.
  3. Pregnant women and lactating mothers are entitled to a ‘ take-home ration’ of 1600 calories per day during pregnancy and for six months thereafter.
  1. 1 and 2
  2. 2 only
  3. 1 and 3
  4. 3 only

Sol. Answer is (b). Statements 1 and 3 are wrong.

According to Chapter VI of the NFS Act, women of eighteen years of age or above to be head of household for purpose of issue of ration cards. So statement 2 is correct. Allocation of subsidised food grains ONLY to people below poverty line is not mentioned in the bill.

The NFS Act entitled every pregnant woman and lactating mother to meal, free of charge, during pregnancy and six months after child birth. It specified a calorie count of 600 calories.

It would look tough to remember figures like "600 calories" etc., but that forms the basis of the Act's deliverables and hence should be remembered.

View image of the nutrition table here

Download NFS Act PDF here


26. India enacted The Geographical Indications of Goods(Registration and Protection) Act, 1999 in order to comply with the obligations to

  1. ILO
  2. IMF
  3. UNCTAD
  4. WTO

Sol. Answer is (d).

Geographical Indications of Goods are defined as that aspect of industrial property which refer to the geographical indication referring to a country or to a place situated therein as being the country or place of origin of that product. Typically, such a name conveys an assurance of quality and distinctiveness which is essentially attributable to the fact of its origin in that defined geographical locality, region or country. Under Articles 1 (2) and 10 of the Paris Convention for the Protection of Industrial Property, geographical indications are covered as an element of IPRs. They are also covered under Articles 22 to 24 of the Trade Related Aspects of Intellectual Property Rights (TRIPS) Agreement, which was part of the Agreements concluding the Uruguay Round of GATT negotiations.

India, as a member of the World Trade Organization (WTO), enacted the Geographical Indications of Goods (Registration & Protection)Act, 1999 has come into force with effect from 15th September 2003.

View a summary image here

Download the GI Act, 1999, English here

Download the GI Act, 1999, Hindi here

Read more on WTO and GI tags in PDF, here


27. Consider the following statements :

  1. In India, State Governments do not have the power to auction non-coal mines.
  2. Andhra Pradesh and Jharkhand do not have gold mines.
  3. Rajasthan has iron ore mines.
  1. 1 and 2
  2. 2 only
  3. 1 and 3
  4. 3 only

Sol. Answer is (d). Statements 1, 2 are wrong.

State governments can auction non-coal mines. In 2017 itself, Rajasthan, Odisha, Karnataka, Madhya Pradesh, Andhra Pradesh, Chhattisgarh and Jharkhand auctioned mining leases for limestone, iron ore, diamond, and gold in 21 rounds this financial year and would receive revenue from miners during the life of the mining lease, which usually is 50 years.

Sluggish commodity prices had initially held back momentum in the auctioning of mining leases after the January 2015 amendment to the Mines and Minerals (Development and Regulation) (MMDR) Act, 1957, but it gathered pace later. Another 150-200 mines across minerals were put up for auction in 2017-18. The law was amended to replace the administrative allocation of mining leases with auctions.

Jharkhand indeed has gold mines. Example : Kunder Kocha mines. Rajasthan has iron ore mines, hence statement 3 is correct.

During 2017-18, mineral production was reported from 32 States/Union Territories of which the bulk of value of mineral production (excluding fuel and atomic minerals) of about 93.65% was confined to 10 States.

Rajasthan is in leading position, in terms of estimated value of mineral production in the country and had the share of 20.26% in the national output. Next in order was Odisha with a share of 17.77% followed by Andhra

Pradesh (9.45%), Chhattisgarh (8.80%), Karnataka (7.83%), Telangana (6.06%), Gujarat (5.66%), Uttar Pradesh (4.98%), Maharashtra (4.67%) and Bihar (3.77%) in the total value of mineral production.

View support images here

Read a PDF on Gold in India - here

Read more on Minerals in India - here


28. With reference to digital payments, consider the following statements:

  1. BHIM app allows the user to transfer money to anyone with the UPI- enabled bank account.
  2. While a chip pin debit card has four factors of authentication, BHIM app has only two factors of authentication.

Which of the statements given above is/are correct ?

  1. 1 only
  2. 2 only
  3. both 1 and 2
  4. neither 1 and 2

Sol. Answer is (a). Statement 2 is wrong.

BHIM app is a universal platform allowing for transfer of money to any Unified Payments Interface bank account in India. So statement 1 is correct.

BHIM app's key feature is its three factor authentication. Hence statement 2 is wrong. The three levels of authentication are : (1) The app binds with a device’s ID and mobile number, (2) A user needs to sync whichever bank account (UPI or non-UPI enabled) in order to the conduct transaction, and (3) When a user sets up the app they are asked to create a pin which is needed to log into the app. Further, the UPI pin, which a user creates with their bank account is needed to go through with the transaction.

Refer image here

Download PDF on BHIM and digital payments - here

Liked Analysis? Write a lovely Testimonial! Thanks :)


29. Among the following cities, which one lies on a longitude closest to that of Delhi ?

  1. Bengaluru
  2. Hyderabad
  3. Nagpur
  4. Pune

Sol. Answer is (a). - Bengaluru!

A good question that innovatively checked your understanding of Indian geography!

Delhi -77.1025° E

Bengaluru - 77.5946° E

Hyderabad - 78.4867° E

Nagpur - 79.0882° E

Pune - 73.8567° E

Do check the map - here


30. International Labour Organisation’s Conventions 138 and 182 are related to

  1. Child Labour
  2. Adaptation of agricultural practices to global climate change
  3. Regulation of food prices and food security
  4. Gender parity of the workplace

Sol. Answer is (a). Child labour.

You could have eliminated options (b) and (c) directly, as they are not related to ILO (International Labour Organisation).

Convention 138 of the International Labour Organisation is concerned with the minimum age of employment.

Convention 182 of the International Labour Organisation is concerned with the prohibition and immediate action for elimination of worst kinds of child labour.

We covered this in Prelims Cracker Course, among many others.

Reference image here

PIB reference of news - here


31. Regarding Money Bill, which of the following statements is not correct ?

  1. A bill shall be deemed to be a Money Bill if it contains only provisions relating to imposition, abolition, remission, alteration or regulation of any tax.
  2. A Money Bill has provisions for the custody of the Consolidated Fund of India or the Contingency Fund of India.
  3. A Money Bill is concerned with the appropriation of moneys out of the Contingency Fund of India.
  4. A Money Bill deals with the regulation of borrowing of money or giving of any guarantee by the Government of India.

Sol. Answer is (c). Option (c) is incorrect hence it is our answer.

Option (a) correctly mentions the definition of a Money Bill (as per Article 110(1)(a))

Option (b) also correctly mentions the definition of a Money Bill (as per Article 110(1)(c))

Option (d) also correctly mentions the definition of a Money Bill (as per Article 110(1)(b))

Article 110(1) of the Constitution defines a Money Bill as :

A Bill is deemed to be a Money Bill if it contains only provisions dealing with all or any of the following matters, namely:

(a) the imposition, abolition, remission, alteration or regulation of any tax;

(b) the regulation of the borrowing of money or the giving of any guarantee by the Government of India, or the amendment of the law with respect to any financial obligations undertaken or to be undertaken by the Government of India;

(c) the custody of the Consolidated Fund or the Contingency Fund of India, the payment of moneys into or the withdrawal of moneys from any such fund;

(d) the appropriation of moneys out of the Consolidated Fund of India;

(e) the declaring of any expenditure to be expenditure charged on the Consolidated Fund of India or the increasing of the amount of any such expenditure;

(f) the receipt of money on account of the Consolidated Fund of India or the public account of India or the custody or issue of such money or the audit of the accounts of the Union or of a State;

or

(g) any matter incidental to any of the matters specified in sub-clauses (a) to (f).

(2.) A Bill is not deemed to be Money Bill by reason only that it provides for the imposition of fines or other pecuniary penalties, or for the demand or payment of fees for licences or fees for services rendered, or by reason that it provides for the imposition, abolition, remission, alteration or regulation of any tax by any local authority or body for local purposes….

The Contingency Fund of India is established under Article 267(1) of the Indian Constitution. It is used in the nature of natural disasters and unforesable events. The fund is held by the Finance Secretary on behalf of the President of India.

Check out our Downloads page for Full constitution of India PDF, and hundreds more - HERE


32. With reference to the election of the President of India, consider the following statements :

  1. The value of the vote of each MLA varies from State to State.
  2. The value of the vote of MPs of the Lok Sabha is more than the value of the vote of MPs of the Rajya Sabha.

Which of the statements given above is/are correct ?

  1. 1 only
  2. 2 only
  3. Both 1 and 2
  4. Neither 1 and 2

Sol. Answer is (a). Statement 2 is incorrect

Statement 1 is correct.

The formula for deciding the value of each vote of a MLA is the ratio of the total population of the said state with the total number of elected members of the legislative assembly which is further divided by 1000. So each state has a different value of MLA vote.

However this is not true in case of the Lok Sabha and the Rajya Sabha.

Reference images here .

Full ECI document on Presidential Elections, here


33. In the Indian Context, what is the implication of ratifying the ‘ Additional Protocol’ with the ‘International Atomic Energy Agency (IAEA)’ ?

  1. The civilian nuclear reactors come under IAEA safeguards.
  2. The military nuclear installations come under the inspection of IAEA.
  3. The country will have the privilege to buy uranium from the Nuclear Suppliers Group (NSG).
  4. The country automatically becomes the member of NSG.

Sol. Answer is (a).

The "Additional Protocol" of the IAEA significantly increases its ability to verify the peaceful use of all nuclear material in India. So civil nuclear reactors fall under IAEA safeguards. India will not become a member of the NSG by agreeing with the additional protocol. India will not have the rights to buy uranium from the NSG.

As of May 2017, Additional Protocols are in force with 129 States and Euratom. Another 19 States have signed an Additional Protocol but have yet to bring it into force. One State provisionally applies an Additional Protocol to its comprehensive safeguards agreement, pending its entry into force.

Under the Additional Protocol, the IAEA is granted expanded rights of access to information and locations in the States. For States with a CSA (comprehensive safeguards agreement), the Additional Protocol aims to fill the gaps in the information reported under a CSA. By enabling the IAEA to obtain a much fuller picture of such States’ nuclear programmes, plans, nuclear material holdings and trade, the Additional Protocol increases the IAEA’s ability to provide much greater assurance on the absence of undeclared nuclear material and activities in those States.

Refer images - here

Read the PDF of IAES Additional Protocol, here

Read Ready Reckoner on it, here

34. Consider the following countries:

  1. Australia
  2. Canada
  3. China
  4. India
  5. Japan
  6. USA

Which of the above are among the ‘ free- trade partners’ of ASEAN ?

  1. 1, 2, 4 and 5
  2. 3, 4, 5 and 6
  3. 1, 3, 4 and 5
  4. 2, 3, 4 and 6

Sol. Answer is (c).

ASEAN has five free trade agreements with six countries - China, Japan, Republic of Korea, India, and Australia and New Zealand. So the correct answer is (c). This was a tough one, as it is very specific.

The objectives of these FTAs are to:

(i) Act as a catalyst in strengthening ASEAN’s trade and economic relationship with China, Japan, Republic of Korea, India, and Australia and New Zealand among ASEAN Member States and the Republic of Korea by creating a large free trade area to provide economies of scale for companies;

(ii) Enhance economic competitiveness of ASEAN and these countries as well as increase the standard of living of the people through the progressive liberalization and promotion of trade in goods and services and the creation of a transparent, liberal, and facilitative investment regime.

(iii) Enhance economic integration of ASEAN and these East Asian countries and provide a building block for the creation of Regional Comprehensive Economic Partnership Agreement and thereby assisting in reducing the development gaps.

(iv) The creation of a huge free market covering and these East Asian countries will bring dynamic benefits to the region since it will attract more foreign direct investments, which will subsequently create more jobs and facilitate the transfer of advanced technology.

ASEAN does not have a free trade agreement with the US or Canada. ASEAN countries are not to be confused with ASEAN free trade partners.

Check our reference images - here

Download reference PDF - here


35. With reference to the ‘Global Alliance for Climate Smart Agriculture (GACSA)’, which of the following statements is/are correct ?

  1. GACSA is an outcome of the climate summit held in Paris in 2015.
  2. Membership of GACSA does not create any binding obligations.
  3. India was instrumental in the creation of GACSA.

Select the correct answer using the code given below:

  1. 1 and 3 only
  2. 2 only
  3. 2 and 3 only
  4. 1, 2 and 3

Sol. Answer is (b). Only statement 2 is correct.

GACSA is an inclusive, voluntary and action-oriented multi-stakeholder platform (promoted by the FAO) on Climate-Smart Agriculture (CSA).

Statement 1 is wrong as it was launched in 2014 at the UNSG Climate Summit, New York. Statement 3 is also wrong as India did not play that role. Till May 2018, of the 236 member entities, India was not one.

Key GACSA events :

Launch of the Alliance at the UNSG Climate Summit, 24 September 2014 (source : http://www.fao.org/3/a-bl861e.pd )

(2) First working meeting of the Alliance at FAO Head Quarters in Rome on 17-18 December 2014

(3) First meeting of the GACSA Strategic Committee in Abu Dhabi on 11 March 2015

Second meeting of the GACSA Strategic Committee at FAO head Quarters in Rome on 14 October 2015

Third meeting of the GACSA Strategic Committee in Rotterdam on 09 May 2016

First Annual Forum of GACSA at FAO Head Quarters in Rome on 14 – 17 June 2016

Full reference PDF document - here


36. Which of the following is/are the aim/aims of “Digital India” Plan of the Government of India ?

  1. Formation of India’s own internet companies like China did.
  2. Establish a policy framework to encourage overseas multinational corporations that collect Big Data to build their last data centres within our national geographical boundaries.
  3. Connect many of our villages to the Internet and bring Wi-Fi to many of our schools, public places nd major tourist centres.

Select the correct answer using the code given below:

  1. 1 and 2 only
  2. 3 only
  3. 2 and 3 only
  4. 1, 2 and 3

Sol. Answer is (b). Only 3 is correct.

Statement 1 is wrong. The Digital India plan has not spoken of building Indian internet companies "like China did". This direct comparison is not present.

( from http://digitalindia.gov.in/content/vision-and-vision-areas

The vision of Digital India programme is to transform India into a digitally empowered society and knowledge economy.

Vision areas are :

1) Digital Infrastructure as a Core Utility to Every Citizen

2) Governance and Services on Demand

3) Digital Empowerment of Citizens

Statement 2 is wrong as this was not a part of the original vision but was added later (end 2017) with RBI demanding local data hosting and storage. That debate is separarely raging!

Refer images here


37. Consider the Following Pairs:

Which of the pairs given above are correctly matched ?

  1. 1 and 2
  2. 1 and 4
  3. 2 and 3
  4. 3 and 4

Sol. Answer is (b). All these were in news for the wrong reasons.

Aleppo is a city in Syria, it is the capital of the Aleppo governate. Continuous strife brought this in news frequently. First, identify this connection as you cannot get it wrong! So options (c) and (d) are eliminated.

Kirkuk is a city in Iraq, serving as the capital of the Kirkuk Governorate.

Mosul is a city in Northern Iraq. This was the site for the massive battle against the ISIS.

Mazar-i-sharif is a town in Afghanistan. This is easy to remember due to the fact that following the September 11 attacks in 2001, it was the first Afghan city to fall to the U.S.-backed Northern Alliance (United Front). The Taliban's defeat in Mazar quickly turned into a rout from the rest of the north and west of Afghanistan. More recently, the Taliban have scaled up their attacks here.

So correct pairing is 1 and 4.

Refer image here


38. In the Federation established by The Government of India Act of 1935, residuary power were given to the

  1. Federal Legislature
  2. Governor General
  3. Provincial Legislature
  4. Provincial Governors

Sol. Answer is (b).

This Act, said to be the longest Act (British) of Parliament ever enacted, aimed to established a “Federation of India” made up of British Indian Provinces and Indian states which might accede to form the union. The

Governor General had special responsibilities regarding certain specified subjects. Residuary legislative powers were vested in the Governor-General.

Refer image - here

Download PDF of GoI Act, 1935 - here

Liked Analysis? Write a lovely Testimonial! Thanks :)


39. Consider the following statements:

  1. The Speaker of the Legislative Assembly shall vacate his/her office if he/she ceases to be a member of the Assembly.
  2. Whenever the Legislative Assembly is dissolved, the Speaker shall vacate his/her office immediately.

Which of the statements given above is/are correct ?

  1. 1 only
  2. 2 only
  3. Both 1 and 2
  4. Neither 1 and 2

Sol. Answer is (a). Statement 2 is wrong.

Statement 1 is correct.

Article 179 of the Indian Constitution states that any member holding office as speaker or deputy speaker of an assembly will have to immediately vacate his office if she/he ceases to be a member of the Assembly.

Whenever the Assembly is dissolved, the Speaker shall not vacate his office before the first meeting of the Assembly after the dissolution.

Refer image here


40. Which of the following reflects the most appropriate relationship between law and liberty ?

  1. If there are more laws, there is less Liberty.
  2. If there are no laws there is no liberty.
  3. If there is liberty, laws have to be made by people.
  4. If laws are changed too often liberty is in danger.

Sol. Best answer is (b).

1. Some are of the opinion that liberty is the ability to do whatever they want. Some think that Law is what restricts one’s exercise of personal liberty. But the two are not at odds with one other; rather, they complement one another.

2. True liberty only comes as a result of established law, and the only established law that exists is the law established by the one and only Law-giver. Always it is the law that guarantees freedom and liberty to individuals. In the absence of law, the powerful will simply usurp what others deserve.

3. The Indian Constitution provides to its citizen Liberty through Article 21, and varied freedoms including speech and expression, movement, assembly, profession etc. via Article 19. The constitution of the USA in a series of Amendments provided to its citizens freedom of press, speech, association, assembly, gun-rights etc. Thus in most constitutional democracies and monarchies freedoms and liberties are guaranteed in the constitution.

4. The judiciary is the watchdog of these constitutions and applies the law to protect and liberty of the people. This liberty is a combination of positive and negative liberty; it not only lets the individual develop and fulfil their potential but also protects them from interference of external agents.

5. Laws are required for liberty of all citizens. So automatically if there are no laws there is no liberty. It should be noted that it is a misconception that no laws would mean absolute liberty as every individual would be able to act to his/her wishes which will not ensure liberty for another citizens.

Option (a) is wrong because it may not always be so if the laws are carefully designed.

Option (c) is wrong as total liberty (to do anything) is chaos and not liberty.

Option (d) is wrong as the Indian Constitution has been amended more than 100 times, yet we have liberty of various types guaranteed to us.


41. Consider the following statements:

  1. No criminal proceeding shall be instituted against the Governor of a state in any court during his terms of office.
  2. The emoluments and allowances of the Governor of a state shall not be diminished during his terms of office.

Which of the statement given above is /are correct?

  1. 1 only
  2. 2 only
  3. Both 1 and 2
  4. Neither 1 nor 2

Sol. Answer is (c). A standard question from the Constitution and Polity section.

As per Article 361 of the Indian constitution, no criminal proceedings can be started or continued against the President, or the Governor of any State in any court, including the Supreme Court during his/her term of office. No court can even issue arrest warrants for the President as well as any Governor.

The emoluments, allowance and privileges may be determined by the Parliament by law. The governer's (Emoluments, Allowances and Privileges) Act, 1982 was amended in 2008. According to the act, the emoluments and allowances of Governor cannot be diminished during his term of office.

Statements 1 and 2 both are correct.

Refer images - here

Download the full constitution of India - here


42. The well known painting “Bani Thani “ belongs to the:

  1. Bundi School
  2. Jaipur School
  3. Kangra School
  4. Kishangarh School

Sol. Answer is (d). A classic and tough question from Art and Culture.

Bani Thani is an Indian painting painted by Nihal Chand from the school of Kishangarh. It depicts a graceful and elegant woman (often compared to Mona Lisa). Bani Thani was believed to be the mistress of King Sawant Singh, and later became one of his wives. Within the Ashta Nayika classification system of heroines, the Bani Thani is identified as the Vasakasajja Nayika type, with the element of Sringara rasa being main.

The Bundi School is an important school of the Rajasthani style of Indian miniature painting that lasted from the 17th to the end of the 19th century in this princely state. One of the earliest examples of the Bundi Paintings is the Chunar Ragamala painted in 1561. Bundi paintings emphasized on hunting, court scenes, festivals, processions, life of nobles, lovers, animals, birds and scenes from Lord Krishna's life.

Kangra painting is the pictorial art of Kangra, named after Kangra, Himachal Pradesh, a former princely state, which patronized the art. It became prevalent with the fading of Basohli school of painting in mid-18th century.

Jaipur school : The finest works, dating from the reign of Pratāp Singh, are sumptuous in effect and include some splendid portraits and some large paintings of the sports of Krishna.

Refer all images here


43. What is “Terminal High Altitude Area Defense (THAAD)”,sometimes seen in the news?

  1. An Israeli Radar System
  2. India’s indigenous anti-missile programme
  3. An American anti-missile system
  4. A defence collaboration between Japan and South Korea

Sol. Answer is (c). A sitter!

Terminal High Altitude Area Defense (THAAD) is an American anti-ballistic missile defense system designed to stop and down short, medium and intermediate-range ballistic missiles. The THAAD interceptor carries no warhead, but relies on its kinetic energy of impact to destroy the incoming missile. A kinetic energy hit minimizes the risk of exploding conventional-warhead ballistic missiles, and the warhead of nuclear-tipped ballistic missiles will not detonate on a kinetic-energy hit! It was in news throughout in 2017 due to South Korea's decision to install it, leading to huge protests from China (and threats from North Korea).

The THAAD system is being designed, built, and integrated by Lockheed Martin Space Systems acting as prime contractor.

[ Total number of sitters that all students must crack : 11 questions. In Set A, these were questions numbered 6 (MDR), 7 (NSG-NPT), 15 (ATM-NPCI), 18 (Yemen-Sudan), 24 (Two state solution), 26 (GIS-WTO), 43 (THAAD), 58 (Petya), 66 (IoT), 90 (Catalonia), and 92 (Right to Privacy). No student should have missed these ones ]

Refer images here


44. With reference to culture history of India,consider the following statements:

  1. Most of the Tyagaraja Kritis are devotional songs in praise of Lord Krishna.
  2. Annamacharya and Tyagaraja are contemporaries.
  3. Annamacharya kirtnas are devotional songs in praise of Lord Venketshwara.

Which of the statement is given above are correct?

  1. 1 only
  2. 2 only
  3. Both 1 and 2
  4. Neither 1 nor 2

Sol. Answer is (b). A very specific and hence tough question from Art and Culture!

Tyagaraja Kritis are mostly devotional songs in the praise of Lord Rama. So statement 1 is incorrect.

Statement 3 is incorrect because Tyagaraja flourished during the 19th century however Annamacharya was born much earlier (1408).

So, the only correct option left is (b).

Background : Tyagaraja (1767 – 1847) composed many devotional compositions, mostly in Telugu and in praise of Lord Rama. Five of his compositions called the Pancharatna Kritis ("five gems") are often sung in programs in his honour. Annamācārya (Annamayya) (1408 – 1503) was a Hindu saint and the earliest known Indian musician to compose songs called "sankirtanas" in praise of the god Venkateswara, a form of Vishnu. He is the first known composer in carnatic music.

Refer images here


45. Which of the following are regarded as the main feature of the “Rule of Law”?

  1. Limitations of powers
  2. Equality before law
  3. People’s responsibility to the Government
  4. Liberty and Civil Rights

Select the correct answer using the code given below:

  1. 1 and 3 only
  2. 2 and 4 only
  3. 1, 2 and 4 only
  4. 1, 2, 3 and 4

Sol. Answer is (c).

"Rule of Law" is a strong concept that underlies all modern societies' governance and functioning.

Broadly speaking, it will refer to the absence of arbitrariness, presence of clearly defined laws and systems, and a functioning judiciary to check the excesses of the Executive.

(1) All institutions' powers are limited, and there is no one with unlimited or abstract powers. The absence of abstract power - no institution can punish someone without reason - is a core principle.

(2) Equality before the law ensures that all citizens are treated equally.

(3) People's responsibility to the government is NOT a core principle, as they are responsible to themselves, the society and the nation on a bigger scale. It is the government that is answerable to the people. So, Statement 3 is wrong.

(4) Liberty and civil rights are the basic idea in "rule of law".

Hence, option (c) is correct.


46. Which one of the following statements correctly describes the meaning of legal tender money?

  1. The money which is tendered in courts of law to defray the fee of legal cases
  2. The money which a creditor is under compulsion to accept in settlement of his claims
  3. The bank money in the form of cheques, drafts, bill of exchange, etc.
  4. The metallic money in circulation in a country

Sol. Answer is (b).

Legal tender is any medium of payment which the legal system of a nation will recognize as valid for meeting a financial obligation. For example paper currency and coins. So the Indian Rupee is India's de-facto legal tender currency and also for neighbours like Nepal and Bhutan. (but Nepalese Rupee is generally not considered legal tender in India).

While a person who is not a pre-existing creditor cannot be compelled to accept legal tender as payment for goods and services, legal tender payment has to be accepted compulsorily for settlement of claims by pre-existing creditors. Options (a) and (c) are wrong as personal checks etc. are usually not legal tender. Option (d) may be right in certain cases, but Option (b) is best here.


47. If a commodity is provided free to the public by Government,then

  1. The opportunity cost is zero.
  2. The opportunity cost is ignored.
  3. The opportunity cost is transferred from the consumers of the product to the tax paying public.
  4. The opportunity cost is transferred from the consumers of the product to the Government.

Sol. Answer is (c).

A subsidy is a benefit given to an individual, business or institution, usually by the government. It is usually in the form of a cash payment or a tax reduction. All subsidies involve an opportunity cost. Government will have to forego (give up) investments in other sectors of the economy in order to provide subsidy. At the end of the day, the burden usually lies on the taxpayer, as government revenues come from taxpayers only (many of whom will also enjoy the subsidies).

If the government provides any commodity at "no cost" to the public, it is doing so on the tax money it collects from the tax payers, which could have been used to directly benefit them. The opportunity cost is directly transferred from the consumers (as they don't pay) to the tax paying public.

Between (c) and (d), we think (c) is better as ultimately taxpayers fund the government, not the other way round. And taxpayers lose out on services they might have got through investments by government now lost in subsidies.

Check reference image here

Download a primer on concepts of Subsidies - here


48. Increase in absolute and per capita real GNP do not connote a higher level of economic development,if

  1. Industrial output fails to keep pace with agricultural output.
  2. Agricultural output fails to keep pace with industrial output.
  3. Poverty and unemployment increases.
  4. Imports grow faster than exports.

Sol. Answer is (c).

Option (a) is irrelevant as the mutual connection of industrial and agricultural output does not answer the question. Same logic holds for option (b). You may have thought that since more than 50% of our workforce is engaged in agriculture, hence option (b) is correct. But without the exact rates of growth (output) being known, any conclusions may not be made.

Option (c) talks about an increase in poverty and unemployment which indicates that a large part in the rise of GNP was from economic activity carried out abroad, or it did not have any direct links with local jobs generation etc. So this is a possible answer.

Option (d) talks about imports growing faster than exports which could indicate a rise in GDP rather than GNP. That would make any conclusion not possible.

Remember : The GNP represents the final value of goods and services produced by domestically owned means of production (using domestic labor and resources), and GDP is the final value of goods and services produced within a given country's border. Some part of a nation's GNP is earned overseas, while some domestic production is added to GDP only.


49. Consider the following statements:

Human capital formation as a concept is better explained in terms of a process which enables

  1. Individuals of a country to accumulate more capital.
  2. Increasing the knowledge,skills level and capacities of the people of the country.
  3. Accumulation of tangible wealth.
  4. Accumulation of intangible wealth.

Which of the statements given above is/are correct?

  1. 1 and 2
  2. 2 only
  3. 2 and 4
  4. 1, 3 and 4

Sol. Answer is (c). Statements 1, 3 are wrong.

Human capital formation does not refer to currency, capital or tangible income. It is a more abstract term which refers to increase in the quality and skill levels and education of the general populace. These are all intangible (not visible / not countable) wealth concepts.

If 1 is obviously wrong, then options (a) and (d) are eliminated. Between (c) and (d), note that 4 is definitely correct (intangible). Hence, answer is (c).


50. Despite being a high saving economy, capital formation may not result in significant increase in output due to

  1. Weak administrative machinery
  2. Illiteracy
  3. High population density
  4. High capital-output ratio

Sol. Answer is (d).

Definition : Capital output ratio (COR) is the amount of capital needed to produce one unit of output. So if it is high, that means we are putting in a lot of money to get some output - a bad situation.

Eliminate options (b) and (c) outright. Why? Illiteracy is too broad and general a term. Many illiterates too have been trained into jobs that made them productive (in labour-intensive sectors). High population density can be used as a strength (like Deng Xiaoping did in 1978 in China).

We are left with (a) and (d).

Option (a) is too broad and vague. But (d) tells us that if COR is high, then we are unable to produce adequate output even after huge capital input (from high savings in the economy).

Liked Analysis? Write a lovely Testimonial! Thanks :)




51. After the Santhal rising subsided,what was/were the measure/measures taken by the colonial Government?

  1. The territories called ‘Santhal Paraganas’ were created.
  2. It become illegal for a Santhal to transfer land to a Non Santhal.

Select the correct answer using a code given below:

  1. 1 only
  2. 2 only
  3. Both 1 and 2
  4. Neither 1 nor 2

Sol. Answer is (c).

The Santhal Pargana District was created in 1885 after taking 5000 square miles from the existing districts of Bhagalpur and Birbhum. It was to be a "non regulation district" and normal administrative and judicial processes were not to apply there. The village of Dumka became the administrative centre, away from non-Santhal influence. The Santhal village was recognised as the official smallest unit of government in the SP district.

It was a measure taken by the British colonial rulers as they wanted to avoid another uprising like the Santhal Revolt. The revolt has reference to the establishment of the permanent land settlement of 1793, and ongoing resentment with moneylenders.

It became illegal for a Santhal to transfer land to a non-Santhal, as the British wanted to protect their land rights.

Hence both 1 and 2 are correct. Answer is (c).

Read more in a PDF, here


52. Economically, one of the result of the British rule in India in the 19th century was the

  1. Increase in the exports of Indian handicrafts
  2. Growth in the number of Indian owned factories
  3. Commercialization of Indian agriculture
  4. Rapid increase in urban population

Sol. Answer is (c). Of all the options, this is the best.

Option (b) is factually wrong, and (d) is not that big an outcome (even today, Indian urban population is 34% of the total).

Under the British rule, manufacture and export of Indian handicrafts greatly reduced as the policies made by the British discouraged Indians from selling their products overseas, by levying very high duties. The number of Indian owned factories was miniscule, Indians generally didn't own anything.

Option (c) is correct. The British rule had pronounced and profound economic impact on India, as their policies turned India into a purely "colonial economy" to serve the needs of the British economy, not local needs. They did everything to commercialise the agriculture in India.

1. Commercialisation of Agriculture means that the agricultural crops and goods are produced by farmers for sale in the market and not for self consumption.

2. Since Indian industrial scene was not developed, Indian agri products were serving industries in Britain, France, Belgium and many other European countries of eighteenth century.

3. Agricultural products needed by the British industries or those that could fetch cash commercial gain to the British in the European or American market, were promoted.

4. Production of cotton in India was pushed, to provide raw and good quality cotton to the cotton-textile industries of Britain, growing fast. Hence, cotton growing area increase in India and its production increased manifold. Indigo and tea, coffee plantation were encouraged because these had a rich market abroad. Plantations were controlled by the British.

5. Jute received attention as jute products got a ready market in America and Europe.

6. Lot of policies were needed to effect such production - (a) The new land tenure system (permanent settlement and Ryotwari Settlement) had made agricultural land a freely exchangeable commodity. The Permanent settlement by giving ownership right to the zamindars created a class of wealthy landlords; they could make use of this ownership right by sale or purchase of land. (b) Crops like cotton, jute, sugarcane, ground nuts, tobacco etc. were increasingly cultivated.

7. The commercialization of agriculture was a forced process for Indian peasants. Farmers had to grow commercial crop on a specified tract of his land under the oppression of planters.

8. The misery of Indian farmers was further enhanced became the population of India was increasing every year, fragmentation of land was taking place because of the increasing pressure on land and modern techniques of agricultural production were not introduced in India.

9. The net result of the commercialization of Indian agriculture was that most of Indian farmers failed to produce even enough to provide them even two square meals a day.

10. The famous Indigo revolt in 1859 symbolised the tensions. Later, such rampant commercialization of Indian agriculture manifested in a series of famines claiming millions of lives.


53. If President of India exercises his power as provided under Article 356 of the Constitution in respect of a particular State, then

  1. The assembly of the State is automatically dissolved.
  2. The powers of the Legislature of that State shall be exercisable by or under the authority of the Parliament.
  3. Article 19 is suspended in that state.
  4. The President can make laws relating to that state.

Sol. Answer is (b). A very simple question from polity!

Article 356 has Provisions that in case of failure of constitutional machinery in State, the President may declare that the powers of the Legislature of the State shall be exercisable by or under the authority of Parliament. So the correct answer is (b).

Refer image - here

Download the full Constitution PDF - here


54. Consider the following pairs :

Which of the pairs given above is/are correct?

  1. 1 only
  2. 1 and 2
  3. 3 only
  4. 2 and 3

Sol. Answer is (a). Another one from Art and Culture. Two of the three have received GI Tags already (Uppada Jamdani sarees and Sujni embroidery).

Sujni embroidery originated in Bihar, not Maharashtra. So options (b) and (d) are incorrect. Uppada Jamdani Sarees (a handicraft) are orignally from Andhra Pradesh so option (d) is also ruled out.

Hence answer is (a) - 1 only.


55. In which of the following area can GPS technology be used ?

  1. Mobile phones operation
  2. Banking operations
  3. Controlling the power grids

Which of the pairs given above is/are correct?

  1. 1 only
  2. 2 and 3 only
  3. 1 and 3 only
  4. 1, 2 and 3

Sol. Answer is (d).

GPS is already extensive used for mobile operations.

Banking operations can be simplified by GPS as it can increase banks reach to remote areas where otherwise it would have been every difficult for banks to operate. GPS's accurate time helps everyday activities such as banking, mobile phone operations, and the control of power grids by allowing well synchronized hand-off switching.

Only option left is (d).

Background : The Global Positioning System (GPS) is a satellite-based radionavigation system owned by the US government and operated by the US Air Force. It provides geolocation and time information to a GPS receiver anywhere on or near the Earth where there is an unobstructed line of sight to four or more GPS satellites. The GPS does not need the user to transmit any data, and operates independently of any telephonic or internet reception, though these can enhance the usefulness of the GPS. The GPS system can be blocked by the US government (as with the Indian military in 1999 during the Kargil War). Hence, others are setting up their own GPS versions. Examples : (1) The Russian Global Navigation Satellite System (GLONASS), (2) China's BeiDou Navigation Satellite System due to achieve global reach in 2020, (3) European Union Galileo positioning system, and (4) India's NAVIC.

View image here


56. Consider the following statements:

  1. The Reserve Bank of India manages and services Government of India Securities but not any State Government Securities.
  2. Treasury bills are issued by the Government of India and there are no treasury bills issued by the State Governments.
  3. Treasury bills offers are issued at a discount from the par value.

Which of the statements given above is/are correct ?

  1. 1 and 2 only
  2. 3 only
  3. 2 and 3 only
  4. 1, 2 and 3

Sol. Answer is (c). Statement 1 is wrong.

Statement 1 is wrong because the RBI also manages many state government securities.

In terms of Sec. 21A (1) (b) of the Reserve Bank of India Act, 1934, the RBI may, by agreement with any State Government undertake the management of the public debt of that State. Accordingly, the RBI has entered into agreements with 29 State Governments and one Union Territory (UT of Puducherry) for management of their public debt. Under Article 293(3) of the Constitution of India (Under section 48A of Union territories Act, in case of Union Territory), a State Government has to obtain the permission of the Central Government for any borrowing as long as there is any outstanding loan that the State Government may have from the Centre.

So options (a) and (d) are wrong (both have statement 1).

Statement 2 is correct as Treasury Bills are focused for short term (maturities less than one year) borrowing by the Central Government only

Statements 2 and 3 are correct. Answer is (c).

A reference image is here .


57. Consider the following statements:

  1. The Earth’s magnetic fields has reserved every few hundred thousand years.
  2. When the Earth was created more than 4000 millions years ago, there was 54% oxygen and no carbon dioxide.
  3. When living organism originated, they modified the early atmosphere of the Earth.

Which of the statements given above is/are correct ?

  1. 1 only
  2. 2 and 3 only
  3. 1 and 3 only
  4. 1, 2 and 3

Sol. Answer is (c). Both 1 and 3 are correct.

1. It is accepted now that for nearly half of Earth's 460 crore year history, the atmosphere contained almost no oxygen. Cyanobacteria or blue-green algae became the first microbes to produce oxygen by photosynthesis, perhaps as long ago as 350 crore years ago (approximately). So 3 is correct. Check options - Option (a) is gone.

2. Flipping of polarity or Geomagnetic Reversal : If geologic history repeats itself, Earth’s magnetic poles should eventually swap places. Based on the magnetic fingerprints locked into ancient rocks, we know that over the last 20 million years, magnetic north and south have flipped roughly every 200,000 to 300,000 years (this rate has not been constant over the planet’s lifetime, though). The last of these major reversals occurred about 780,000 years ago, although the Poles do wander around in between these larger flips. (What’s more, climate change seems to be shifting Earth’s geographic poles.)

3. A geomagnetic reversal is a change in a planet's magnetic field such that the positions of magnetic north and magnetic south are interchanged, while geographic north and geographic south remain the same. The Earth's field has alternated between periods of normal polarity, in which the main direction of the field was the same as the present direction, and reverse polarity, in which it was the opposite. These periods are called "chrons".

4. A small complete reversal (the Laschamp event) occurred only 41,000 years ago during the last glacial period. That reversal lasted only about 440 years with the actual change of polarity lasting around 250 years. During this change the strength of the magnetic field weakened to 5% of its present strength.

5. Scientists estimate that past polar flips have been rather sluggish, with north and south migrating to opposite positions over thousands of years. This is both good and bad if you’re concerned about how a geomagnetic reversal will affect life on Earth.

A very nicely framed question that checks the basics of Geography and Science related facts.

View reference images here


58. The terms “WannaCry , Petya and Eternal Blue” sometimes mentioned in the news recently are related to

  1. Exoplanets
  2. Cryptocurrency
  3. Cyber attacks
  4. Mini Satellites

Sol. Answer is (c). This questions was a sitter!

All three - WannaCry, Petya and EternalBlue - were in news as these are related to Cyber attacks.

[ Total number of sitters that all students must crack : 11 questions. In Set A, these were questions numbered 6 (MDR), 7 (NSG-NPT), 15 (ATM-NPCI), 18 (Yemen-Sudan), 24 (Two state solution), 26 (GIS-WTO), 43 (THAAD), 58 (Petya), 66 (IoT), 90 (Catalonia), and 92 (Right to Privacy). No student should have missed these ones ]

Refer an image here


59. With references to the circumstances in Indian agriculture, the concept of “Conservation Agriculture” assumes significance. Which of the following fall under the Conservation Agriculture?

  1. Avoiding the monoculture practices
  2. Adopting minimum tillage
  3. Avoiding the cultivation of plantation crops
  4. Using crop residues to cover soil surface
  5. Adopting spatial and temporal crop sequencing/crop rotations

Select the correct answer using the code given below:

  1. 1, 3 and 4
  2. 2, 3, 4 and 5
  3. 2, 4 and 5
  4. 1, 2, 3 and 5

Sol. Answer is (c).

Statement 2 has to be there, as minimum tillage is a key principle in "Conservation Agriculture" or CA. Hence option (a) is eliminated. Similarly, Statement 4 is correct (crop residue mulching). So option (d) is eliminated. Statement 3 is wrong as no such restriction is known. Hence, option (a) is eliminated. Best answer is (c).

Conservation agriculture (CA) technologies involve minimum soil disturbance, permanent soil cover through crop residues or cover crops, and crop rotations for achieving higher productivity. In India, efforts to develop, refine and disseminate conservation-based agricultural technologies have been underway for nearly two decades and made significant progress since then even though there are several constraints that affect adoption of CA. Particularly, tremendous efforts have been made on no-till in wheat under a rice-wheat rotation in the Indo-Gangetic plains.

1. Conservation agriculture (CA) can be defined by a statement given by the Food and Agricultural Organization as “a concept for resource-saving agricultural crop production that strives to achieve acceptable profits together with high and sustained production levels while concurrently conserving the environment”.

2. It has three key principles that producers (farmers) can use.

The First Principle is practicing minimum soil disturbance which is essential to maintaining minerals within the soil, stopping erosion, and preventing water loss from occurring within the soil. In the past agriculture has looked at soil tillage as a main process in the introduction of new crops to an area. It was believed that tilling the soil would increase fertility within the soil through mineralization that takes place in the soil. But it can cause severe erosion and crusting which leads to a decrease in soil fertility. Today tillage is seen as destroying organic matter that can be found within the soil cover. No-till farming has caught on.

The Second Principle is about managing the top soil to create a permanent organic soil cover can allow for growth of organisms within the soil structure. This growth will break down the mulch that is left on the soil surface. The breaking down of this mulch will produce a high organic matter level which will act as a fertilizer for the soil surface. If CA practices were used done for many years and enough organic matter was being built up at the surface, then a layer of mulch would start to form. This layer helps prevent soil erosion from taking place and ruining the soil's profile or layout. The presence of mulching also reduce the velocity of runoff and the impact of rain drops thus reducing soil erosion and runoff. Crop residue mulching can be done effectively.

The Third Principle is the practicing diverse crop rotations or crop interactions. It is best as a disease control against other preferred crops and does not allow pests such as insects and weeds to be set into a rotation with specific crops. Rotational crops will act as a natural insecticide and herbicide against specific crops. Establishing crops in a rotation allows for an extensive buildup of rooting zones which will allow for better water infiltration.

Organic molecules in the soil break down into phosphates, nitrates and other beneficial elements which are thus better absorbed by plants. Plowing increases the amount of oxygen in the soil and increases the aerobic processes, hastening the breakdown of organic material. Thus more nutrients are available for the next crop but, at the same time, the soil is depleted more quickly of its nutrient reserves.

Reference images available here

Read more - PDF available here


60. The term “sixth mass extinction/sixth extinction” is often mentioned in the news in the context of discussion of

  1. Widespread monoculture practices in agriculture and large scale commercial farming with indiscriminate use of chemicals in many part of the world that may result in the loss of good native ecosystems.
  2. Fears of possible collision of a meteorite with the Earth in the near future in the manner it happened 65 million years ago that caused the mass extinction of many species including those of dinosaurs.
  3. Large scale cultivation of genetically modified crops in many part of the world and promoting their cultivation in other parts of the world which may cause the disappearance of good native crop plants and the loss of food biodiversity.
  4. Mankind’s overexploitation/misuse of natural resources, fragmentation/loss of natural habitats, destruction of ecosystems, pollutions and world climate change.

Sol. Answer is (d). A fairly simple and straight question.

There are two kinds of extinctions that species can undergo.

1. Background Extinction : when species go extinct through natural selection during gradual environmental change or competition between species

2. Mass Extinction : when large numbers of species go extinct together due to very unusual, environmentally catastrophic conditions. Usually unrelated to their normal adaptations to their environment. The world has

seen at least 5 mass extinctions so far

(i) End Ordovician, 444 million years ago, 86% of species lost

(ii) Late Devonian, 375 million years ago, 75% of species lost

(iii) End Permian, 251 million years ago, 96% of species lost

(iv) End Triassic, 200 million years ago, 80% of species lost

(v) End Cretaceous, 66 million years ago, 76% of all species lost

3. A “biological annihilation” of wildlife in recent decades means a sixth mass extinction in Earth's history is under way due to purely anthropogenic reasons.

Reference image here

Can download a PDF on Mass Extinctions - here


61. With reference to the Indian Regional Navigation Satellite System (IRNSS) , consider the following statements :

  1. IRNSS has three satellites in geostationary and four satellites in geosynchronous orbits.
  2. Geosynchronous covers entire India and about 5500 sq. Km beyond its border.
  3. India will have its own satellite navigation system with full global coverage by the middle of 2019.

Which of the following statements given above is/are correct?

  1. 1 only
  2. 1 and 2 only
  3. 2 and 3 only
  4. none

Sol. Answer is (a). Statements 2, 3 are wrong.

IRNSS covers India and a region extending 1,500 km around it. So assuming that is the radius, it covers approximately ( 22,50,000 x Pi ) sq. km. Anyway, the 5500 sqkm figure given is too low (Indian landmass itself is 32 lac sqkm). Clearly Statement 2 is wrong. So options (b) and (c) are incorrect.

The constellation consists of 7 active satellites. Three of the seven satellites in constellation are located in geostationary orbit (GEO) and four in inclined geosynchronous orbit (GSO). Statement 1 is correct.

Reference image can be seen here

Reference PDF on IRNSS - here


62. Consider the following phenomenon:

  1. Light is affected by gravity.
  2. The universe is constantly expanding.
  3. Matter wraps its surrounding space- time.

Which of the above is/are prediction/predictions of Albert Einstein’s General Theory Of Relativity, often discussed in media ?

  1. 1 and 2 only
  2. 3 only
  3. 1 and 3 only
  4. 1, 2 and 3

Sol. Answer is (d). All three statements are correct.

General relativity theory published by Albert Einstein in 1915 is the current description of gravitation in modern physics.

The theory of general relativity is the basis of current cosmological models of a consistently expanding universe. The bending of light by gravity can lead to the phenomenon of gravitational lensing, in which multiple images of the same distant astronomical object are visible in the sky. According to the relativist John Archibald Wheeler studies on the theory of general relativity, spacetime tells matter how to move; matter tells spacetime how to curve. So all statements are correct.

PDF on General Theory of Relativity - here

PDF on Special Theory of Relativity - here


63. With reference to the Genetically Modified mustard (GM Mustard) developed in India, consider the following statements:

  1. GM Mustard has the genes of the soil bacterium that give the plant the property of pest-resistance to a wide variety of pests.
  2. GM Mustard has the genes that allow the plant cross-pollination and hybridization.
  3. GM mustard has been developed jointly by the IARI and Punjab Agricultural University.

Which of the statements given above is/are correct ?

  1. 1 and 3 only
  2. 2 only
  3. 2 and 3 only
  4. 1, 2 and 3

Sol. Answer is (b).

The trick is to see that 3 is wrong - it was developed by Delhi University. Hence, only option (b) is correct!

News - May 12, 2017 -

"The application on GM Mustard presented by Deepak Pental of South Campus (Delhi University) was appraised at the GEAC meeting held on Thursday. The GEAC has recommended the approval of GM Mustard to the environment ministry. Earlier, a sub-committee had appraised the safety document for GM Mustard", said Amita Prasad, chairperson of the GEAC. Pental, ex-vice chancellor of the Delhi University and a well-known genetics scientist, had filed the application for commercial release of the GM Mustard in December 2015. The GEAC had subsequently set up a sub-committee to examine the safety aspect of the use of transgenic variety of the mustard. The sub-committee had last year given its safety clearance while noting that the GM Mustard is safe for human consumption and environment.

DMH (Dhara Mustard Hybrid)-11 is genetically modified variety of Herbicide Tolerant (HT) mustard. It was developed by Centre for Genetic Manipulation of Crop Plants at Delhi University under Government sponsored project. It was created by using “barnase/barstar” technology for genetic modification by adding genes from soil bacterium that makes mustard self-pollinating plant. It contains three genes viz. Bar gene, Barnase and Barstar sourced from soil bacterium. That bar gene makes the plant resistant to herbicide named Basta.

GM mustard doesn't have a significant pest resistance property.

We had covered this at length in the first session of Power of 10 - view here


64. Consider the following pairs:

Which of the pairs given above is/are correctly matched ?

  1. 1 and 3 only
  2. 2 only
  3. 2 and 3 only
  4. 1, 2 and 3

Sol. Answer is (b). A very direct question!

Blockchain is an open ledger that can safely record transactions between two parties efficiently and in a verifiable method. Blockchain was invented by Satoshi Nakamoto in 2008 to serve as the basis of the cryptocurrency bitcoin. So matching in 2 is correct, but it does not help much as only option (a) gets eliminated.

The Belle II experiment is a particle physics experiment designed to study the properties of B mesons. The experiment is currently being commissioned at the SuperKEKB accelerator complex in Japan. So matching in 1 is wrong.

CRISPR stands for Clustered Regularly Interspaced Short Palindromic Repeats, and is a family of DNA sequences in bacteria and archaea. CRISPR - Cas9 is an RNA-guided DNA endonuclease enzyme associated with adaptive immunity system. So the matching in 3 is wrong.

If you remembered that CRISPR is related to genetic science, then you'd realise that 3 is wrong, so three options get eliminated in one shot, leaving only option (b).


65. Which of the following statements best describes “carbon fertilization” ?

  1. Increased Plant growth due to increased concentration of carbon dioxide in the atmosphere
  2. Increased temperature of the earth due to increased concentration of carbon dioxide in the atmosphere
  3. Increased acidity of oceans as a result of increased concentration of carbon dioxide in the atmosphere
  4. Adaptation of living beings on Earth to the climate change brought about by the increased concentration of carbon dioxide in the atmosphere

Sol. Answer is (a).

Carbon fertilization effect states that the larger amount of carbon dioxide in the atmosphere that has resulted from rising anthropogenic emissions should help the growth of plants, which use carbon dioxide during photosynthesis. So correct option is (a).

This was a direct question.

Reference image is here


66. When the alarm of your smartphone rings in the morning, you wake up and tap it to stop the alarm which causes your geyser to be switched on automatically. The smart mirror in your bathroom shows the day’s weather and also indicates the level of water in your overhead tank. After you take some groceries from your refrigerator for making breakfast, it recognises the shortage of stock in it and places and order for the supply of fresh grocery items. When you step out of your house and lock the door, all lights, fans, geysers and AC machines get switched off automatically. On your way to office, your car warns you about traffic congestion ahead and suggests an alternative route, and if you are late for a meeting, it sends a message to your office accordingly.

In the context of emerging communication technologies, which one of the following terms best applies to the above scenario?

  1. Border Gateway Protocol
  2. Internet Of Things
  3. Internet Protocol
  4. Visual Private Network

Sol. Answer is (b). What a lengthy, and utterly simple question!

The Internet of Things (IoT) is the network where machines talk with each other. It comprises physical devices, vehicles, home appliances and other items embedded with electronics, software, sensors, actuators, and connectivity which enables these things to interact with each other.

Of all the options, option (b) should have been clear to you.

Here are the reference images - click

Liked Analysis? Write a lovely Testimonial! Thanks :)


67. With reference to solar power production in India, consider the following statements :

  1. India is largest in the world in the manufacture of silicon wafers used in photovoltaic units.
  2. The solar power tariffs are determined by the Solar Energy Corporation of India.

Which of the following statements given above is/are correct ?

  1. 1 only
  2. 2 only
  3. Both 1 and 2
  4. Neither 1 nor 2

Sol. Answer is (d).

Statement 2 is wrong as the process of competitive bidding is conducted to arrive at the tariffs.

Solar Energy Corporation of India Ltd. (SECI) is a CPSU under the administrative control of the Ministry of New and Renewable Energy (MNRE), set up on 20th Sept, 2011 to facilitate the implementation of JNNSM and achievement of targets set therein. It is the only CPSU dedicated to the solar energy sector. It was originally incorporated as a section-25 (not for-profit) company under the Companies Act, 1956. To achieve the target of achieving 100 gigawatt (GW) of installed solar energy capacity by 2022, the Central government has invited tenders for 2,000 MW of solar power projects connected to the inter-state transmission system. The Solar Energy Corporation of (SECI) would sign 25-year power purchase agreements (PPAs) with the winning bidders and sell the power to electricity distribution utilities (discoms). The ceiling tariff has been set at Rs 2.93 per unit (news dated 2018).

Also, India does not figure in the top 3 makers of silicon wafers used in PV units.

Best answer is (d).


68. WThe staple commodities of export by the English East India Company from Bengal in the middle of the 18th century were

  1. Raw cotton, oil-seeds and opium
  2. sugar, salt, zinc and Lead
  3. Copper, silver, gold, spices and tea
  4. Cotton, silk, saltpetre and opium

Sol. Answer is (d).

East India company had a monopoly of trade in cotton and silk piece goods, indigo, and saltpeter produced in India. EIC also traded opium grown in India with Chinese tea. Bengal port was the main hub of trade for the company as it bought raw products at very low rates and sent them to England or sell them to neighbouring countries like China.

Cotton was a key item of export. Opium was sent to China, and other places. Silk was produced in large quantity in Bengal, and saltpetre used for explosives and gunpowder etc. too was exported.

An interesting PDF on East India Company - here


69. Which one of the following is a very significant aspect of the Champaran Satyagraha ?

  1. Active all-India participation of lawyers, students and women in the National Movement
  2. Active involvement of Dalit and Tribal communities of India in the National Movement
  3. Joining of peasant unrest to India’s National Movement
  4. Drastic decrease in the cultivation of plantation crops and commercial crops

Sol. Answer is (c).

The Champaran Satyagraha of 1917 was Mahatma Gandhi’s first "Satyagraha" - a polite insistence for truth. The peasants (bhumihars) of the Champaran and other areas of North Bihar were growing the Indigo under the tinakathia system. Gandhi therefore set up voluntary organizations to improve the economic and educational conditions of the people. It not only marked the beginning of Satyagraha but functioned as a link between various regional farmer's unrest to the national freedom movement.

Options (a), (b) and (d) are wrong.

An interesting PDF on the Champaran Satyagraha - here


70. Who among the following were the founders of the “Hind Mazdoor Sabha” established in 1948?

  1. B. Krishna Pillai, E.M.S. Namboodiripad and K.C George
  2. Jayaprakash Narayan, Deen Dayal Upadhyay and M.N Roy
  3. C.P Ramaswamy Iyer, K. Kamaraj and Veeresalingam Pantulu
  4. Ashok Mehta, T.S Ramanujan and G.G Mehta

Sol. Answer is (d).

The Hind Mazdoor Sabha was founded in West bengal on 29 December 1948, by socialists and independent unionists. Its founders included Basawon Singh (Sinha), R.S. Ruikar, Ashok Mehta, Maniben Kara, T.S. Ramanujam, V.S. Mathur, G.G. Mehta.

A direct question which is fact-based.


71. With reference to the religious practices in India, the “Sthanakvasi” sect belongs to:

  1. Buddhism
  2. Jainism
  3. Vaishnavism
  4. Shaivism

Sol. Answer is (b).

Sthanakavasi is a sect of Svetambara Jainism founded by a merchant named Lavaji in 1653 AD. One of its tenants is the rejection of idol worship as a way to achieve Moksha.

The sect is essentially a reformation of the one founded on teachings of Lonka, a fifteenth-century Jain reformer. Sthānakavāsins accept thirty-two of the Jain Agamas, the Śvētāmbara canon. Śvētāmbarins who are not Sthānakavāsins are mostly part of the Murtipujaka sect.

Read the PDF on Essentials of Jainism - here


72. With reference to the cultural history of India, consider the following statements :

  1. White marble was used in making Buland Darwaza and Khankah at Fatehpur Sikri.
  2. Red Sandstone and marble were used in making Bara Imambara and Rumi Darwaza in Lucknow.

Which of the following statements given above is/are correct ?

  1. 1 only
  2. 2 only
  3. Both 1 and 2
  4. Neither 1 nor 2

Sol. Answer is (d). neither 1 nor 2

1. Buland Darwaza or the "Gate of victory", was built in 1601 A.D. by Akbar. It is made of red and buff sandstone, decorated by white and black marble and is higher than the courtyard of the mosque.

2. Akbar-e-Azam ascended the throne of India in 1556. In the next decade or so, he not only consolidated his empire but also expanded it. Akbar had everything a monarch could ask for except an heir. To pray for one, he undertook the journey from Agra to the village of Sikri where the Sufi saint, Salim Chishti, lived in his hospice. The saint blessed the emperor, and a son was born to Akbar on August 30, 1569. The joyful father named him Salim after the saint, and according to Professor Ali Nadeem Rezavi of the Department of History at Aligarh Muslim University, “decided to heap on this city the resources of a vast empire.”

3. Architectural accomplishments : He built Sikri, later renamed Fatehpur Sikri, or the city of victory. It is an architectural delight, but it is the Buland Darwaza, or ‘gate of magnificence’, that has held many enthralled. The construction of the Buland Darwaza was inspired by Timurid architecture. Along with Humayun’s Tomb, its monumentality reflects its Central Asian origins.

4. Bara Imambada is an imambada complex in Lucknow, India, built by Asaf-ud-Daula, Nawab of Awadh, in 1784. It is also called the Asafi Imambara. Bara means big, and imambara is a sacred hall built for the purpose of Azadari. The Bara Imambara is among the grandest buildings of Lucknow. The roof of Imambara is made up from the rice husk which make this Imambara a unique building.

5. The Rumi Darwaza in Lucknow is an imposing gateway which was also built under the patronage of Nawab Asaf-Ud-daula in 1784. It is an example of Awadhi architecture.

6. Asaf-ud-Daula’s rule saw a devastating famine, which created an economic crisis. The residents of Awadh were self-respecting people, so instead of handing out dole, the Nawab started a food-for-work programme. The famous Asafi Imambara, or Bara Imambara, of Lucknow was built to give employment and revenue to the public. Instead of stones and marble, brick and lime were used. Stucco ornamentation (gajkari) was used to decorate the monuments, giving it a deep relief effect even on flat walls. Mother of pearl and shells deposited in lake beds were used in the stucco ornamentation to give a shine finer than marble.

7. The local masons cleverly used the brick, with its small size and thickness, to form remarkably fine details on the wall and column surfaces. It’s a testimony to their skill that they could adapt lowly material to such wonderful effect: balusters were imitated in clay supported on iron rods. Similarly, pottery was used for roof finials and ornaments.

This could be directly solved from an article that appeared in The Hindu newspaper.


73. Which one of the following foreign travellers elaborately discussed about Diamonds and diamond mines in India ?

  1. Francois Bernier
  2. Jean-Baptiste Tavernier
  3. Jean de Thevenot
  4. Abbe Barthelemy Carre

Sol. Answer is (b).

Jean-Bapiste Tavernier was a French gem merchant and a remarkable man, who made six journeys to Persia and India. His works include Les Six Voyages de Jean Baptiste Tavernier, Ecuyer, Baron d’Aubonne, en Turquie, en Perse, et aux Indes. Chez Olivier de.Varennes, 1st ed. Paris 1675, where he discussed about diamond mines in India.

An illustration made by him can be viewed here

1. François Bernier was a French physician and traveller, who briefly was the personal physician to Mughal prince Dara Shikoh (the eldest son of the Mughal emperor Shah Jahan), and after Dara Shikoh's demise, was attached to the court of the Mughal emperor Aurangzeb.

2. Jean de Thévenot was an accomplished polyglot, skilled in Turkish, Arabic and Persian. He was in Mughal Empire of India for thirteen months between 1663 to 1667, and crossed the country by Golconda to Masulipatam, returning overland to Surat.

3. Abbe Barthelemy Carre was another Frenchman who visited India and the Near East, 1672 to 1674, when the French were struggling to establish colonies in this region.


74. With reference to the Indian History, who among the following is a future Buddha, yet to come to save the world ?

  1. Avalokiteshvara
  2. Lokesvara
  3. Maitreya
  4. Padmapani

Sol. Answer is (c).

According to Buddhist traditions, Maitreya is a bodhisattva who will appear on Earth in the future, achieve complete enlightenment, and teach the pure dharma. According to scriptures, Maitreya will be a successor to the present Buddha. The prophecy of the arrival of Maitreya refers to a time in the future when the dharma will have been forgotten by most people - that is when Maitreya will appear on Earth in the future, achieve complete enlightenment, and teach the pure dharma. Maitreya will be a successor to the present Buddha, Gautama Buddha (also known as Śākyamuni Buddha.

Avalokiteśvara is the bodhisattva who embodies the compassion of all Buddhas. This bodhisattva is shown differently in various cultures as either female or male. In Chinese Buddhism, Avalokiteśvara has become the somewhat different female figure Guanyin. In Sanskrit, Avalokiteśvara is also referred to as Padmapāni ("Holder of the Lotus") or Lokeśvara ("Lord of the World").

Thus, answer clearly is (c) only.

View reference images here .


75. Which of the following statements do not apply to the system of Subsidiary Alliance introduced by Lord Wellesley ?

  1. To maintain a large standing army at others expense
  2. To keep India safe from Napoleonic danger
  3. To secure a fixed income for the company
  4. To establish British paramountcy over the Indian states

Sol. Answer is (c).

In the subsidiary alliance, princely states rulers were not allowed to make any negotiations and treaty with any other ruler. They were also not allowed to have an independent armed force. They were to be protected by the East India Company but had to pay for the subsidiary forces that the company was to maintain for protection. Indian rulers had to maintain British troops in their state. The ruler would dismiss any Europeans other than Britishers in his state.

Read a PDF on the topic here


76. Which of the following led to the introduction of English Education in India ?

  1. Charter Act of 1813
  2. General Committee of Public Instruction, 1823
  3. Orientalist and Anglicist Controversy

Select the correct answer using the code given below:

  1. 1 and 2 only
  2. 2 only
  3. 1 and 3 only
  4. 1, 2 and 3

Sol. Answer is (b). Statement 2 is correct, as it directly led to the final introduction of English Education in India.

The Charter Act of 1813 did not clarify the objectives of education and the methods for improvement of literature of the learned natives in India. The Charter Act had given stress on allotting the one lakhs rupees only; no specific regulations were granted for establishing the schools and colleges in India. So statement 1 is not correct. The Charter Act of 1813 made a State system of Education in India officially.

In 1823, the Governor-General-in Council appointed a “General Committee of Public Instruction”, which had the responsibility to grant the one lakh of rupees for education. That committee consisted of 10(ten) European members of which Lord Macaulay was the president. The committee decided to spend major portions from the grant for the improvement of oriental literature. A controversy arose between the Orientalists and Anglicists (promoters of British education) over which method to be followed. That controversy by itself did not "introduce English education in India". Statement 3 is wrong.

Statement 2 is correct. As a president of General Committee of Public Instruction, Lord Macaulay wrote a minute on 2nd February 1835, where he made the conclusion regarding the controversy. Lord Macaulay stressed the implementation of the English language as a medium of instruction through his minute. He said that English was the best medium of instruction, as that this would enable the emergence of a class of people in the Indian society, who would be well versed in English language, western ideology, taste and opinion. This class would serve as a medium of contact with the great mass of Indian people who were culturally different from the English.

In his minute, Macaulay criticized the oriental learning as “a single shelf of good European library was worth the whole native literature of India and Arabic”. Macaulay believed that English should be introduced because it is a language of the ruling class and also the familiarity has been gained by higher classes of Indians. Lord William Bentinck accepted Macaulay’s minute or opinions towards the lanquage of education for India on 7th March 1835.

What a fascinating sequence of events it was!

Liked Analysis? Write a lovely Testimonial! Thanks :)


77. Which of the following is an Artificial Lake ?

  1. Kodaikanal (Tamil Nadu)
  2. Kolleru (Andhra Pradesh)
  3. Nainital (Uttarakhand)
  4. Renuka ( Himachal Pradesh)

Sol. Answer is (a).

Kodaikanal Lake is a manmade lake located in the Kodaikanal city in Dindigul district in Tamil Nadu, India. Sir Vere Henry Levinge was instrumental in creating the lake in 1863, amidst the Kodaikanal town which was developed by the British.

Kolleru lake (AP), Nainital lake (UK) and Renuka lake (HP) natural lakes. Of these 3, Kolleru and Renuka are threatened wetlands requiring immediate attention.

Refer images - here


78. With the reference to the Pradhan Mantri Kaushal Vikas Yojana, consider the following statements :

  1. Charter Act of 1813
  2. It is flagship scheme of the Ministry of Labour and Employment.
  3. It, among other things, will also impart training in soft skills, entrepreneurship, financial and Digital Literacy.
  4. It aims to align the competencies of the unregulated workforce of the country to the National Skill Qualification Framework.

Which of the statements given above is/are correct ?

  1. 1 and 3 only
  2. 2 only
  3. 2 and 3 only
  4. 1, 2 and 3

Sol. Answer is (c). Statement 1 is wrong.

PMKVY is a flagship scheme under the ministry of Skill Development and & Entrepreneurship. That was a giveaway. So options (a) and (d) are gone.

The Short Term Training imparted at PMKVY Training Centres (TCs) is expected to benefit candidates of Indian nationality who are either school/college dropouts or unemployed. Apart from providing training according to the National Skills Qualification Framework (NSQF), TCs shall also impart training in Soft Skills, Entrepreneurship, Financial and Digital Literacy.

Individuals with prior learning experience or skills shall be assessed and certified under the Recognition of Prior Learning (RPL) component of the Scheme. RPL aims to align the competencies of the unregulated workforce of the country to the NSQF.

Statements 2 and 3 are correct.

Read the full PDF on National Policy on Skill Development and Entrepreneurship - here


79. In 1920, which of the following changed its name to “Swarajya Sabha” ?

  1. All India Home Rule League
  2. Hindu Mahasabha
  3. South Indian Liberal Federation
  4. The Servants of India Society

Sol. Answer is (a).

The Indian Home Rule movement was a movement in British India on the lines of Irish Home Rule movement and other home rule movements, which lasted around two years between 1916–1918 and set the stage for the independence movement under the leadership of Annie Besant and B. G. Tilak. In 1920, the All India Home Rule League elected Mahatma Gandhi as its president. In a year, the body merged into the Indian National Congress to form a united Indian political front. On 24 October, 1920, Gandhiji wrote on All-India Home Rule League reconstituted as "Swarajya Sabha".

A tough question based on factual trivia!


80. Which among the following events happened earliest?

  1. Swami Dayanand established Arya Samaj
  2. Dinabandhu Mitra wrote Neeldarpan
  3. Bankim Chandra Chattopadhyay wrote Anandmath
  4. Satyendranath Tagore became the first Indian to succeed in the Indian Civil Services Examination

Sol. Answer is (b).

Arya Samaj was founded on 10 April 1875.

Dinabandhu Mitra wrote Neeldarpan in the year 1860.

Anandamath is a Bengali fiction, written by Bankim Chandra Chattopadhyay and published in 1882.

Satyendranath was selected for the Indian Civil Service in June, 1863.

A nice way of asking questions based on dates!

Travel through time - a beautiful PDF on various events of past 400 years - here


81. Which of the is/are the possible consequences of heavy sand mining in riverbeds?

  1. Decreased salinity in the water
  2. Pollution of groundwater
  3. Lowering of the water label

Select the correct answer using the code below:

  1. 1 only
  2. 2 and 3 only
  3. 1 and 3 only
  4. 1, 2 and 3

Sol. Answer is (b). Statement 1 is wrong.

Excessive instream sand-and-gravel mining causes the degradation of rivers, which results in pollution of the water beneath the river bed. Sand mining also affects the adjoining groundwater system and the uses that local people make of the river.

Instream mining lowers the stream bottom, which may lead to bank erosion. Depletion of sand in the streambed and along coastal areas causes the deepening of rivers and estuaries, and the enlargement of river mouths and coastal inlets. It may also lead to saline-water intrusion from the nearby sea, if any. The effect of mining is compounded by the effect of sea level rise. Any volume of sand exported from streambeds and coastal areas is a loss to the system.

Excessive instream sand mining is a threat to bridges, river banks and nearby structures. Sand mining also affects the adjoining groundwater system and the uses that local people make of the river.

Statements 2 and 3 are correct.


82. With reference to agriculture soils consider the following statements:

  1. A high content of organic matter drastically reduces its water boiling capacity.
  2. Soil does not play role in the sulphur cycle.
  3. Irrigation over a period of time can contribute to the salinization of some agricultural lands.

Which of the statements given above is/are correct?

  1. 1 and 2 only
  2. 3 only
  3. 1 and 3 only
  4. 1, 2 and 3

Sol. Answer is (b). Statements 1 and 2 are incorrect.

Soil plays a very important role in sulphur cycle. Bacterial oxidation and reduction of sulphur compounds takes place inside the soil. A high content of organic matter increases the water holding capacity of the soil as organic matter has a high affinity towards water. So Statement 1 is wrong. That rules out 3 options in one stroke!

Details : Soil water holding capacity is controlled primarily by the soil texture and the soil organic matter content. Soil texture is a reflection of the particle size distribution of a soil. An example is a silt loam soil that has 30% sand, 60% silt and 10% clay sized particles. In general, the higher the percentage of silt and clay sized particles, the higher the water holding capacity. The small particles (clay and silt) have a much larger surface area than the larger sand particles. This large surface area allows the soil to hold a greater quantity of water. The amount of organic material in a soil also influences the water holding capacity. As the level of organic matter increases in a soil, the water holding capacity also increases, due to the affinity of organic matter for water.

Only correct option left is (b).

Statement 3 is correct.

View supporting image here


83. The Partnership for Action Green Economy (PAGE), a UN mechanism to assist countries transition towards greener and more inclusive economies, emerged at

  1. The Earth Summit on Sustainable Development 2002, Johannesburg
  2. The United Nations Conference on Sustainable Development 2012, Rio De Janeiro
  3. The United Nations Framework Convention on Climate Change 2015, Paris
  4. The World Sustainable Development Summit 2016, New Delhi

Sol. Answer is (b).

1. In 2012, Rio+20 (the United Nations Conference on Sustainable Development) was held in Brazil. The conference’s outcome document entitled The Future We Want was a call to action for governments, business and the UN alike to support countries interested in transition to a green economy.

2. PAGE was created as the UN’s direct response to this call of action. Four UN organizations, including ILO, UN Environment, UNIDO and UNITAR initially joined forces to be able to provide countries with broad and tailored support. UNDP joined the partnership in 2014, further increasing the partnership’s capabilities.

3. The five agencies have previously undertaken joint green initiatives. However, this is the first time that all five partners have come together to coordinate their support, expertise and resources at the national level.

Reference images here


84. “3D Printing” has applications in which of the following?

  1. Preparation of confectionery items
  2. MAnufacture of bionic cars
  3. Automotive industry
  4. Data processing technologies

select the correct answer using the code given below;

  1. 1, 3 and 4 only
  2. 2,3 and 4 only
  3. 1 and 4 only
  4. 1, 2, 3, 4 and 5

Sol. Answer is (d). All statements are correct.

The question is rather poorly crafted, as the statement 1 (Preparation of confectionery items) is not a mainstream application of 3D printing, but a specialised one. Statements 2, 3 and 4 are correct. These are well-known, mainstream applications of 3D printing technology.

So our answer must have 2, 3 and 4.

Statement 5 indicates data processing technologies, which is the first step in 3D printing, not the final output! Actually 5 should not be part of the answer. But there is no such option given (2, 3 and 4). Hence, we settle for option (d) which has 1, 2, 3, 4 and 5.

Read a primer on 3D Printing - here

Liked Analysis? Write a lovely Testimonial! Thanks :)


85. Consider the following statements;

  1. The Barren Island volcano is an active volcano located in the Indian territory.
  2. Barren Island lies about 14o kms cast of Great Nicobar.
  3. The last time the Barren Island volcano erupted was in 1991 and it has remained inactive since then.

Which of the statement given below is/are correct?

  1. 1 only
  2. 2 and 3 only
  3. 3 only
  4. 1 and 3

Sol. Answer is (a). Statements 2 and 3 are wrong.

India’s only active volcano, the Barren Island volcano, is in the Andaman and Nicobar Islands. The Barren Island, about 140 km from Port Blair (not east of Great Nicobar), is surrounded by waters ideal for scuba diving and is home to a wide variety of aquatic life. There have been eruptions every two-three years, the last in February 2016.

View image - here


86. Why is a plant called Prosopis juliflora often mentioned in news?

  1. Its extract is widely used in cosmetics.
  2. It tends to reduce the widely biodiversity in the area in which it grows.
  3. Its extract is used in the synthesis of pesticides.
  4. None of the above

Sol. Answer is (b). Statements 2 and 3 are wrong.

Prosopis juliflora, more commonly known as Junglee kikar, is native to Mexico, South America and the Caribbean. It has become an invasive weed in several countries where it was introduced. It is considered a toxic invader in East Africa, India, Sri Lanka, Hawaii etc.

Refer an image here


87. Consider the following statements:

  1. Most of the world’s coral reefs are in tropical waters.
  2. More than one-third of the world’s coral reefs are located in the territories of Australia, Indonesia and Philippines.
  3. Coral reefs host far more number of animal phyls than those hosted by tropical rainforests.

Which of the statements given above is/are correct?

  1. 1 and 2 only
  2. 3 only
  3. 1 and 3 only
  4. 1, 2 and 3

Sol. Answer is (d). All statements are correct.

1. Coral reefs are highly diverse ecosystems vital to the welfare of human populations throughout the tropical area. Occupying less than one quarter of 1% of the marine environment, coral reefs are home to more than 25% of all known marine fish species

2. 32 of the 34 recognised animal Phyla are found on coral reefs compared to 9 Phyla in tropical rainforests

3. The variety of species living on coral reefs is greater than almost anywhere else in the world. Scientists estimate that more than one million species of plants and animals are associated with coral reef ecosystems.

4. Compared to other ecosystems the tropical rainforests have a very high level of diversity. The warm temperatures and high levels of precipitation make an environment that is suitable for a diverse range of animal species. Many of the Animals that inhabit the tropical Rainforest are species that cannot be found in any other type of environment.

5. But Coral reefs outdo the Tropical Rainforests! They are considered to be some of the most diverse biomes in the world. The high temperatures and protection the reefs provide creates a very suitable environment for living creatures.

6. The immediate threat to coral reefs from climate change is acute; 16% of the world’s reef suffered serious damages during the global bleaching event of 1998.

Refer the image to see the tropical locations on the globe - here

Supporting PDF document - here


88. “Momentum for Change : Climate Neutral Now” is an initiative launched by

  1. The Intergovernmental Panel on Climate Change
  2. The UNEP Secretariat
  3. The UNFCCC Secretariat
  4. The World Meteorological Organization

Sol. Answer is (c). Another fact-based question from climate initiatives!

The Paris Climate Change Agreement’s central aim is to strengthen the global response to climate change by keeping a global temperature rise below 2 degrees Celsius above pre-industrial levels. The UNFCCC secretariat launched its Climate Neutral Now initiative in 2015.

Climate Neutral Now is an initiative launched by the UN Climate Change in 2015, aiming at encouraging and supporting all levels of society to take climate action to achieve a climate neutral world by mid-century, as enshrined in the Paris Agreement adopted the same year.

The initiative therefore invites companies, organizations, governments and citizens to work towards climate neutrality by reducing their climate footprint thanks to a simple 3-steps method, whereby they shall:

Measure their greenhouse gas emissions;

Reduce them as much as possible; and

Compensate those which cannot be avoided by using UN certified emission reductions (CERs).

View image here


89. With the reference to educational institutions during rule in India, consider the following pairs:

Which of the pairs given above is/are correct?

  1. 1 and 2
  2. 2 only
  3. 1 and 3
  4. 3 only

Sol. Answer is (b).

In 1791, during the period of British rule in India, Jonathan Duncan, the representative of the East India Company proposed the establishment of a Sanskrit college for development and preservation of the Sanskrit vangmaya. This was sanctioned by Governor General Lord Cornwallis. So Matching 1 is incorrect.

Aliah University, previously known as the Calcutta Madrasa was established by Warren Hastings in 1780. So Matching 2 is correct.

Fort William College was established by Lord Richard Wellesley in 1800.


90. Consider the following pairs:

Which of the pairs given below above are correctly matched?

  1. 1, 2 and 3
  2. 3 and 4 only
  3. 1 and 3 only
  4. 2 and 4 only

Sol. Answer is (c).

Catalonia is an autonomous region in Spain, much in news due to separatist pronouncements from Spain (resisted by Spain).

Crimea is located south of the Ukrainian region of Kherson. Following a controversial referendum, the official results of which showed majority support for joining Russia, Russian President Vladimir Putin signed a treaty of accession with the self-declared Republic of Crimea, annexing it into the Russian Federation. It became a well-reported issue worldwide, and you should know it. The benefit? Crimea was in two options - hence you can rule out (a) and (d).

Mindanao is the second largest island in the Philippines. A heavy conflict is going on in the Autonomous Region in Muslim Mindanao between Muslim militias and the Philippine government.

Oromias is one of the nine ethnically based regional states of Ethiopia.

Even if you did not know about 3 and 4, the answer was obvious - option (c) - 1 and 3 only.


91. Consider the following events:

  1. The first democratically elected communist party government formed in a State in India.
  2. India’s then largest bank,”Imperial Bank of India” was renamed “State Bank of India” .
  3. Air India was nationalized and became the national carrier.
  4. Goa became a part of independent India.

Which of the following is the correct chronological sequence of the above events?

  1. 4-1-2-3
  2. 3-2-1-4
  3. 4-2-1-3
  4. 3-1-2-4

Sol. Answer is (b).

In 1953, the Government of India passed the Air Corporations Act and purchased a majority stake in the carrier from Tata Sons.

The Imperial Bank of India (IBI) was the oldest and the largest commercial bank of the Indian subcontinent, and was subsequently transformed into State Bank of India in 1955.

In 1957, the CPI won the state elections in Kerala.

Goa was liberated on 19th December 1961.


92. Right to Privacy is protected as an intrinsic part of Right to Life and Personal Liberty.Which of the following in the Constitution of India correctly and appropriately imply the above statements?

  1. Article 14 and the provisions under the 42nd Amendments to the Constitution
  2. Article 17 and the Directive Principles of State policy in Part IV
  3. Article 21 and the freedom guaranteed in Part III
  4. Article 24 and the provision under the 44th Amendments to the Constitution

Sol. Answer is (c).

As already entailed by the Article 21 of the Constitution of India states that “No person shall be deprived of his life or personal liberty except according to procedure established by law”. The right to life therefore includes all those aspects of life which makes a man’s life more meaningful, complete and right to privacy is one such right.

This judgement by the Supreme Court of India in August 2017 was a landmark verdict by a majority of 9-0!

View support images here

Read the verdict here

Liked Analysis? Write a lovely Testimonial! Thanks :)


93. Consider the following:

  1. Area nut
  2. Barley
  3. Coffee
  4. Finger millet
  5. Groundnut
  6. Sesamum
  7. Turmeric

The Cabinet Committee on Economic Affairs has announced the Minimum Support Price for which of the above?

  1. 1, 2, 3 and 7 only
  2. 2, 4, 5 and 6 only
  3. 1, 3, 4, 5 and 6 only
  4. 1, 2, 3, 4, 5, 6 and 7

Sol. Answer is (b).

Barley, finger millet (Ragi), Groundnut (Moongfaali) and sesamum (Till) are covered under the Minimum support price that had been announced by the Cabinet Committee on Economic Affairs.

Reference link - http://pib.nic.in/newsite/PrintRelease.aspx?relid=145856


94. In which of the following States is Pakhui Wildlife Sanctuary located?

  1. Arunachal Pradesh
  2. Manipur
  3. Meghalaya
  4. Nagaland

Sol. Answer is (a). One of the most direct, thought not the easiest, questions.

Pakhui wildlife sanctuary is located in the East Kameng district of Arunachal Pradesh.


95. With the reference to India’s satellite launch vehicles, consider the following statements:

  1. PSLVs launch the satellite useful for Earth resources monitoring whereas GSLVs are designed mainly to launch communication satellites.
  2. Satellites launched by PSLVs appear to remain permanently fixed in the first same position in the sky, as viewed from a particular location on Earth.
  3. GSLV Mk III is a four-staged launch vehicle with the first and third stages using solid rocket motors; and the second and fourth stages using liquid rocket engines.

Which of the statements given below is/are correct?

  1. 1 only
  2. 2 and 3 only
  3. 1 and 2
  4. 3 only

Sol. Answer is (a). Only statement 1 is correct.

PSLV is designed mainly to deliver the “earth-observation” or “remote-sensing” satellites with lift-off mass of up to about 1750 Kg.

The GSLV is designed mainly to deliver the communication-satellites to the elliptical geosynchronous Transfer Orbit (GTO). Due to their geo-synchronous nature, the satellites in these orbits appear to remain permanently fixed in the same position in the sky.

GSLV Mk III is a three-stage heavy lift launch vehicle developed by ISRO. The vehicle has two solid strap-ons, a core liquid booster and a cryogenic upper stage.

The question required specific knowledge about the launch vehicles, a justified expectation, given the frequency with which ISRO has been breaking records and making headlines!


96. With the references to the governance of the public sector banking in India, considering the following statements:

  1. Capital infusion into public sector banks by the Government of India has steadily increased in the last decade.
  2. To put the public sector banks in order, the merger of associate Banks with the parent State Bank of India has been affected.

Which of the statements given above is/are correct?

  1. 1 only
  2. 2 only
  3. Both 1 and 2
  4. Neither 1 nor 2

Sol. Answer is (b). Only Statement 2 is correct.

Statement 1 indicates "a steadily increasing capital infusion over past 10 years". That is wrong. In some years, the amounts dropped. Please check support image here.

Statement 2 is indeed correct. That was the purpose of merging.

Refer image regarding capital infusion amounts, here

Such questions were indeed expected, given the NPA issue in Indian PSB system.


97. Consider the following items:

  1. Cereal grains hulled
  2. Chicken eggs cooked
  3. Fish processed and canned
  4. Newspaper containing advertising material

Which of the above items is/are exempted under GST (Goods and Services Tax)?

  1. 1 only
  2. 2 and 3 only
  3. 1, 2 and 4 only
  4. 1, 2, 3 and 4

Sol. Answer is (a) 1 only.

This was an odd question, as extremely micro level information on GST coverage was asked :)


98. Consider of the following statements:

  1. The Definition of “Critical Wildlife Habitat” is incorporated in the Forest Rights, 2006.
  2. For the first time in India, Baigas have been Habitat Rights.
  3. Union Ministry of Environment, Forest and Climate Change officially decides and declares Habitat Rights for Primitives and Vulnerable Tribal Groups is any part of India.

Which of the statements given below is/are correct?

  1. 1 and 2 only
  2. 2 and 3 only
  3. 3 only
  4. 1, 2 and 3

Sol. Answer is (a).

The power to notify the rules to designate a CWH rests with Ministry of Environment and Forests.

The government of Madhya Pradesh has for the first time recognised the habitat rights of seven villages in Dindori district, mostly inhabited by the Baigas.

State governments can declare habitat rights for primitive and vulnerable tribal groups in any part of India. So, statement 3 is wrong.

Liked Analysis? Write a lovely Testimonial! Thanks :)


99. Consider the following:

  1. Birds
  2. Dust blowing
  3. Rain
  4. Wind blowing

Which of the above plant diseases:

  1. 1 and 3 only
  2. 3 and 4 only
  3. 1, 2 and 4 only
  4. 1, 2, 3 and 4

Sol. Answer is (d). All are correct.

Direct and indirect transmission of plant diseases can happen.

Direct transmission : Disease transmission where the pathogen is carried externally or internally on the seed or planting material like cuttings, sets, tubers, bulbs etc.

Indirect transmission : The pathogen spreading itself by way of its persistent growth or certain structures of the pathogen carried independently by natural agencies like wind, water, animals, insects, mites, nematodes, birds etc. are the different methods of indirect transmissions.

Liked Analysis? Write a lovely Testimonial! Thanks :)


100. With references to organic farming in India, consider the following statements:

  1. “The National Programme for Organic Production”(NPOP) is operated under the guidelines and directions of the Union Ministry Of the Rural Development.
  2. “The Agricultural and Processed and Products Export Development Authority”(APEDA) functions as the Secretariat for the implementation of NPOP.
  3. Sikkim has become India’s first fully organic State.

Which of the following given above is/are correct?

  1. 1 and 2 only
  2. 2 and 3 only
  3. 3 only
  4. 1, 2 and 3

Sol. Answer is (b). 2 and 3 only are right.

Sikkim is India first fully organic state. Hence, statement 3 is correct. So (a) is ruled out.

NPOP is operated under the guidelines and directions of the Ministry of Commerce and Industry. So, 1 is wrong, so option (d) is also ruled out.

According to the scope and opertaional structure of national organic program, APEDA shall function as the Secretariat for the implementation of the NPOP.

Hence, answer is (b).

Reference image here.

Liked Analysis? Write a lovely Testimonial! Thanks :)












Questions and Detailed Solutions are being continuously updated ... refresh and check. Comment and let us know your experience, answers and solutions too!